Está en la página 1de 123

DESIGUALDADES

Problem Solving

Gerard Romo Garrido


Toomates Coolección
Los documentos de Toomates son materiales digitales y gratuitos. Son digitales porque están pensados para ser consultados mediante un
ordenador, tablet o móvil. Son gratuitos porque se ofrecen a la comunidad educativa sin coste alguno. Los libros de texto pueden ser digitales o
en papel, gratuitos o en venta, y ninguna de estas opciones es necesariamente mejor o peor que las otras. Es más: Suele suceder que los mejores
docentes son los que piden a sus alumnos la compra de un libro de texto en papel, esto es un hecho. Lo que no es aceptable, por inmoral y
mezquino, es el modelo de las llamadas "licencias digitales" con las que las editoriales pretenden cobrar a los estudiantes, una y otra vez, por
acceder a los mismos contenidos (unos contenidos que, además, son de una bajísima calidad). Este modelo de negocio es miserable, pues
impide el compartir un mismo libro, incluso entre dos hermanos, pretende convertir a los estudiantes en un mercado cautivo, exige a los
estudiantes y a las escuelas costosísimas líneas de Internet, pretende pervertir el conocimiento, que es algo social, público, convirtiéndolo en un
producto de propiedad privada, accesible solo a aquellos que se lo puedan permitir, y solo de una manera encapsulada, fragmentada,
impidiendo el derecho del alumno de poseer todo el libro, de acceder a todo el libro, de moverse libremente por todo el libro.
Nadie puede pretender ser neutral ante esto: Mirar para otro lado y aceptar el modelo de licencias digitales es admitir un mundo más injusto, es
participar en la denegación del acceso al conocimiento a aquellos que no disponen de medios económicos, y esto en un mundo en el que las
modernas tecnologías actuales permiten, por primera vez en la historia de la Humanidad, poder compartir el conocimiento sin coste alguno, con
algo tan simple como es un archivo "pdf". El conocimiento no es una mercancía.
El proyecto Toomates tiene como objetivo la promoción y difusión entre el profesorado y el colectivo de estudiantes de unos materiales
didácticos libres, gratuitos y de calidad, que fuerce a las editoriales a competir ofreciendo alternativas de pago atractivas aumentando la calidad
de unos libros de texto que actualmente son muy mediocres, y no mediante retorcidas técnicas comerciales.
Este documento se comparte bajo una licencia “Creative Commons 4.0 (Atribution Non Commercial)”: Se permite, se promueve y se
fomenta cualquier uso, reproducción y edición de todos estos materiales siempre que sea sin ánimo de lucro y se cite su procedencia. Todos los
documentos se ofrecen en dos versiones: En formato “pdf” para una cómoda lectura y en el formato “doc” de MSWord para permitir y facilitar
su edición y generar versiones parcial o totalmente modificadas.

¡Libérate de la tiranía y mediocridad de las editoriales! Crea, utiliza y comparte tus propios materiales didácticos

Toomates Coolección Problem Solving:


Geometría Axiomática , Problemas de Geometría (vol.1) , Problemas de Geometría (vol.2)
Introducción a la Geometría , Álgebra , Teoría de números , Combinatoria , Probabilidad
Trigonometría , Desigualdades , Números complejos
Toomates Coolección Llibres de Text (en catalán):
Nombres (Preàlgebra) , Àlgebra , Proporcionalitat , Mesures geomètriques , Geometria analítica
Compendium ACM4 , Combinatòria i Probabilitat , Estadística , Trigonometria , Funcions
Nombres Complexos , Mates amb Excel . Àlgebra Lineal 2n batx. , Geometria Lineal 2n batx.
Càlcul Infinitesimal 2n batx. , Programació Lineal 2n batx.
Toomates Coolección Compendiums:
Ámbito PAU: Catalunya TEC , Catalunya CCSS , Galicia , País Vasco , Portugal A , Portugal B
Ámbito Canguro: Canguro ESP , Cangur CAT , Kangourou FR , Kangaroo USA , Kangaroo UK ,
Känguru AUS
Ámbito Preolímpico: AMC 8 , AMC 10 , AMC 12 , AIME , Archimede , HMMT , Mathcounts
Ámbito Olímpico: IGO , IMO , OME , OMEFL , OMEC , OMEM , OMI , SMT , USAMO , INMO ,
CMO , REOIM

¡Genera tus propias versiones de este documento! Siempre que es posible se ofrecen las versiones editables “MS
Word” de todos los materiales, para facilitar su edición. Descarga en los siguientes enlaces la versión ".doc" de
este documento:
www.toomates.net/biblioteca/Desigualdades.doc

¡Ayuda a mejorar! Envía cualquier duda, observación, comentario o sugerencia a toomates@gmail.com


¡No utilices una versión anticuada! Todos estos documentos se mejoran constantemente. Descarga totalmente
gratis la última versión de estos documentos en los correspondientes enlaces superiores, en los que siempre
encontrarás la versión más actualizada.
Descarga en www.toomates.net/biblioteca/Syllabus.pdf una guía del usuario para la utilización de los materiales
de Toomates Coolección.
Encontrarás muchos más materiales para el aprendizaje de las matemáticas en www.toomates.net
Visita el Canal Youtube de Toomates: https://www.youtube.com/c/GerardRomo
Versión de este documento: 05/01/2023
Índice
1 Las desigualdades fundamentales. →

2 La desigualdad AM-GM. →
Primeros pasos con la desigualdad AM-GM.
La desigualdad AM-GM ponderada.
La desigualdad de Muirhead.
A la búsqueda de la combinación AM-GM adecuada.

3 El cuadro general de las desigualdades entre medias. →

4 La desigualdad Cauchy-Schwarz. →

5 El principio de reordenación. La desigualdad de Chebyshev. →


El principio de reordenación de dos elementos.
El principio de reordenación general.
La desigualdad de Chebyshev.

6 La desigualdad de Jensen. El truco de la recta tangente. →

7 Desigualdades simétricas. Normalización y homogeneización. →

8 Problemas olímpicos y preolímpicos con desigualdades algebraicas. →

9 Desigualdades trigonométricas. →

10 Desigualdades con los lados del triángulo. →


Desigualdad Triangular.
La Transformación de Ravi.

11 Desigualdades geométricas con trigonometría. →

12 Desigualdades con las rectas del triángulo. →

13 Inecuaciones. →

14 Aplicación de las desigualdades en la resolución de ecuaciones. →

Soluciones.→
1 Las desigualdades fundamentales.
1.1 Las desigualdades fundamentales y sus aplicaciones.

Partimos de las dos desigualdades fundamentales:

a) 0  x 2 y 0  x 2  x  0

b) 0  x12  x22  ...  xn2 y 0  x12  x22  ...  xn2  x1  x 2  ...  x n  0

De estas dos anteriores se pueden deducir las dos siguientes: Si a  0 y b  0 ,

c) 2ab  a 2  b2
Sea x  a  b . Entonces 0  (a  b) 2 por el modelo (a), y por tanto:
0  (a  b)2  a 2  b2  2ab  2ab  a 2  b2


d) (a  b)2  2 a 2  b2 
0  (a  b) 2 , nuevamente por el modelo (a), y por tanto

0  (a  b)2  a 2  b2  2ab  a 2  b2  a 2  b2  2 a 2  b2 

1.2
Demuestra que
a b
2 
b a

1.3
1 
Demuestra que  2 para 0  x 
sin x cos x 2

1.4
Demuestra que
1
2a
a

1.5
Demostrar que para todo x , se cumple:
x2  2
2
x2  1
1.6 F
Demostrar que si  1  x, y  1, entonces
x y x y

1  xy 1  x y

1.7 D
Sean a y b dos números reales tales que 1010  a, b  2020 . Demostrar que

a  b 1  1   9
a b 2

OMEC 2021 #1

1.8 D
Si a, b, c, d son números enteros positivos cumpliendo a  b  c  d  63 , determina
max ab  bc  cd 

Cambridge Interview Question

1.9 MF
Determina el valor mínimo de xy  1  x  y  para todos los números reales x , y .
2 2

1 1
(A) 0 (B) (C) (D) 1 (E) 2
4 2
AMC 10A 2021 #9, AMC 12A 2021 #7
2 La desigualdad AM-GM.

Primeros pasos con la desigualdad AM-GM.

2.1 Desigualdad AM-GM con dos variables.

Si a, b  0
ab
ab 
2

y la igualdad solo sucede si a  b .

Demostración. Hemos visto en 1.1c que 2 xy  x 2  y 2


Substituyendo a y b por a y b tenemos
ab
2 ab  2 a b  a  b  ab 
2

2.2 Desigualdad AM-GM en general.


La desigualdad anterior se puede generalizar para n números no negativos a1 ,...an :
a  ...  an
n a ... a  1
1 n
n

y se produce la igualdad si y solo si a1  ...  an .

2.3
Demostrar que si a, b, c  0 , entonces (a  b)(b  c)(c  a)  8 a b c

2.4
Si ai  0 y a1 a2 ... an  1, entonces 1  a1 1  a2 ... 1  an   2n

2.5
a) Demostrar que ab  bc  ac  a 2  b2  c2

b) Demostrar que 3ab  bc  ac   a  b  c   3 a 2  b2  c 2
2

2.6
Demuestra la desigualdad AM-GM con tres números: Si a, b, c  0 , entonces
3 abc 
abc
3
2.7 Problema resuelto.
Sean a1 , a2 ,..., an números reales positivos. Entonces, para cualquier permutación
b1 , b2 ,..., bn de los mismos se cumple
a1 a2 a
  ...  n  n
b1 b2 bn

Solución: En efecto, solo hay que observar que a1 a2 ...an  b1 b2 ...bn , y por tanto,
aplicando la desigualdad AM-GM:
a1 a2 a
  ...  n
b1 b2 bn a a ...a
 n 1 2 n 1
n b1b2 ...bn

2.8
Sea n  1 un número natural. Demuestra que
 n  1
n

n!  
 2 

2.9
Demuestra que si x  y  z  6 entonces x 2  y 2  z 2  12

2.10
1
Sean a, b, c  IR , a 2  b2  c 2  1. Demuestra que  ab  bc  ca  1 .
2

2.11
a 2  b2 a  b
Demuestra que, si a, b  0 , entonces  .
ab 2

2.12
Demostrar que si a, b, c son positivos, entonces:
(b  c  a)(a  c  b)(a  b  c)  abc

2.13
Si a, b, c son tres reales positivos cualesquiera, demostrar que
ab bc ca
  8
c a b

2.14
Demuestra que si a, b, c  0
  
a 2b  b 2 c  c 2 a ab 2  bc 2  ca 2  9a 2b 2 c 2

2.15
Sean a, b, c números reales positivos tales que abc  1 . Demuestra que
a b c
   abc
b c a
2.16
Sean a, b, x, y  0 . Demuestra que ab  xy ax  by   4abxy

2.17
Demostrar que, si a, b, c  0 , abc(a  b  c)  b2c 2  c 2a 2  a 2b2

2.18
 4cos x .
2 2
Determina el mínimo de 4sin x

2.19
a 3 b3 c 3
Suponiendo que a, b, c  0 , demostrar que    abc
bc ac ab

2.20
Sean a, b, c números reales positivos. Demostrar que
a3  b3  c3  a 2b  b2c  c2a

2.21
Sean a, b, c números reales positivos. Demostrar que

a3  b3  c3  ab2  bc2  ca 2  2 a 2b  b2c  c2a 

2.22 D
Sean x, y, z números reales positivos tales que x  y  z  3 . Demostrar que
x3 y3 z3 1
  
 y  2 z  z  2 x  x  2 y  3
2 2 2

2.23 D
Dados a, b, c  0 cumpliendo abc  1, demostrar que
bc ca ab
   a  b  c 3
a b c

2.24 M
Sean a, b, c números reales positivos tales que a  b  c  3 . Demostrar que
a  b  c  ab  bc  ca

Russia MO 2004

2.25 M
   1 , entonces a  1b  1c  1  64
1 1 1
Demuestra que, si
a b c
La desigualdad AM-GM ponderada.

2.26 Desigualdad AM-GM ponderada.


La desigualdad AM-GM se puede generalizar para coeficientes no necesariamente
iguales.
Si a1 , a2 , ..., an y 1 , 2 , ..., n son números reales no negativos, y se cumple
1  2  ...  n  1 , entonces
  
a1 1 a2 2 ... an n  1a1  2a2  ...  n an

o si preferimos una forma más compacta:


n n

 ai i   i ai

i 1 i 1

y se produce la igualdad si y solo si ai  a j para todos los enteros i , j tales que i  0 y


j  0 .
1
La desigualdad AM-GM es un caso particular cuando 1  2  ...  n  .
n

2.27 MF
Sean a, b, c números reales positivos tales que a  b  c  3 . Demuestra que
a bb c c a  1

2.28 MF
Sean a, b, c  0 tales que a  b  c  1 . Demuestra que
a abbcc  abbcca  acbacb  1
La desigualdad de Muirhead.

Sumatorios cíclicos.
Supongamos que trabajamos con tres variables a, b, c .
Las sumas cíclicas son aquellas que se aplican a todas las rotaciones de las tres
variables: a, b, c   b, c, a   c, a, b
 a 2  a 2  b2  c 2
cyc

a b  a b  b c  c a
cyc
2 2 2 2

 abc  abc  bca  cab  3abc


cyc

Sumatorios simétricos.
Son aquellas que se aplican a todas las permutaciones posibles en las tres variables:
Ejemplos:
 a 2   a 2b0c0 a 2b0c0  a 2c0b0  b2a0c0  b2c0a0  c2a0b0  c2b0a0 
sym sym

 a  a 2  b2  b2  c2  c2
2

 a b   a bc
sym
2

sym
2 0
a 2bc 0  a 2cb 0  b2 ac 0  b 2ca 0  c 2 ab0  c 2ba 0 

 a b  a 2 c  b 2 a  b 2 c  c 2 a  c 2b
2

 abc  abc  acb  bac  bca  cab  cba  6abc


sym

Polinomios.
Fijadas unas variables x1 , x2 ,..., xn , y dada una n-tupla a  a1 , a2 ,..., an  , denotaremos
por
a  a1, a2 ,..., an   1  x1a1 x2a2 ...xnan
n! sym
Donde el sumatorio se extiende a todas las permutaciones posibles de x1 , x2 ,..., xn  .

Por ejemplo:
 1 , 1   1 x1 y1  y1x1   1 xy  yx   xy
2! 2
 1 , 1 , 1   xyz  xzy  yxz  yzx  zxy  zyx  xyz
1
3!
 
 2 , 1 , 0   1 x 2 y 1 z 0  x 2 z 1 y 0  y 2 x1 z 0  y 2 z 1 x 0  z 2 x1 y 0  z 2 y 1 x 0 
3!
1
 1
 
 x 2 y  x 2 z  y 2 x  y 2 z  z 2 x  z 2 y  x 2 ( y  z)  y 2 ( x  z)  z 2 ( x  y)
6 6

En particular:
Media aritmética:  1 , 0 , ... , 0  
n  1! x  x  ...  x   1 x  x  ...  x 
1 2 n 1 2 n
n! n
1 1 1  n!

Media geométrica:  , , ... ,   x11/ n x12/ n ... x1n/ n  n x1 x2 ...xn 
n n n  n!
Mayorización de n-tuplas.
Diremos que a1  a2  ...  an mayoriza b1  b2  ...  bn , y escribiremos a   b ,
cuando
a) a1  a2  ...  an  b1  b2  ...  bn
b) Para cada k  1,2,..., n  1 se cumple a1  a2  ...  ak  b1  b2  ...  bk

Está claro que a   a  , y que a   b y b  c  implica a   c 

2.30 Desigualdad de Muirhead.


Se cumple b  a para cierta n-tupla de reales no negativos x1 , x2 ,..., xn si y solo si
b  a  . La igualdad se cumple si y solo si b  y a  son idénticas o bien todos los xi
son iguales.

Ejemplos.
a) ( 2 , 0 )   1,1   x 2  y 2  2 xy
b) ( 3 , 0 , 0 )  (1,1,1)  x 3  y 3  z 3  3xyz
c) ( 5 , 0 )  ( 3 , 2 )  x 5  y 5  x 3 y 2  x 2 y 3
d) ( 2 , 2 , 0 )  ( 2 ,1,1)  x 2 y 2  y 2 z 2  z 2 x 2  x 2 yz  y 2 xz  z 2 xy

Ejemplo.
Puesto que 5,0,0  3,1,1  2,2,1 , tenemos:
a 5  a 5  b5  b5  c5  c5 
a 5b 0 c 0  a 5 c 0 b 0  b 5 a 0 c 0  b 5 c 0 a 0  c 5 a 0 b 0  c 5b 0 a 0 
a 3b1c1  a 3c1b1  b 3 a1c1  b 3c1a1  c 3 a1b1  c 3b1a1 
a 2 b 2 c1  a 2 c 2 b1  b 2 a 2 c1  b 2 c 2 a1  c 2 a 2 b1  c 2 b 2 a1

Es decir:
   
2 a 5  b5  c 5  2 a 3bc  b3 ac  c 3 ab  2abcab  ac  bc
Y finalmente:
a 5  b5  c 5  a 3bc  b3 ac  c 3 ab  abcab  ac  bc 

2.29 MF
Demuestra que si a, b, c son números reales positivos, se cumple:
a2 b2
  a b
b a
Nota biográfica.
Robert Franklin Muirhead (Glasgow, 1860 – 1941) fue un matemático escocés. Estudió
Matemáticas en la Universidad de Glasgow, donde se graduó con excelentes resultados.
Continuó sus estudios en la Universidad Santa Catarina de Cambridge. Allí ganó en
1886 el Premio Smith por su ensayo sobre Las leyes del movimiento de Newton.

Fue elegido miembro de la Sociedad Matemática de Edimburgo en 1884 y fue


nombrado Miembro Honorífico de esta Sociedad en 1912. Publicó varios artículos en
diferentes revistas pero su fama se debe a la llamada desigualdad de Muirhead.

R.F. Muirhead publicó en el año 1903 un artículo sobre el estudio de ciertos métodos
algebraicos aplicables a igualdades y desigualdades de funciones algebraicas simétricas
en n variables. Mientras estudiaba expresiones algebraicas de la forma x1a1 x2a2 ...xnan ,
analizó ciertos polinomios simétricos conteniendo dichas expresiones con el objetivo de
establecer un cierto orden en el espacio de las n-tuplas a1 , a2 ,..., an  satisfaciendo la
condición a1  a2  ...  an .

2.31 MF
Demostrar que, si a, b, c son números reales positivos, se cumple

a3  b3  c3  abc  a  b  c 
1 3

Observación.
Leemos en el documento "Olympiad Inequalities" (Thomas J. Mildorf, December 22,
2005): "Although Muirhead's theorem is a named theorem, it is generally not favorably
regarded as part of a formal olympiad solution. Essentially, the majorization criterion
guarantees that Muirhead's inequality can be deduced from a suitable application of
AM-GM. Hence, whenever possible, you should use Muirhead's inequality only to
deduce the correct relationship and then explicitly write all of the necessary applications
of AM-GM. For a particular case this is a simple matter."

Es decir, la desigualdad Muirhead no forma parte del currículum de teoremas válidos


para las Olimpiadas matemáticas. Nos puede ser útil para garantizar que se cumple
cierta desigualdad, pero debemos ofrecer una demostración basada en AM-GM.
A la búsqueda de la combinación AM-GM adecuada.
Las claves para encontrar una demostración basada en AM-GM son dos:
- Homogeneización de la inecuación.
- Encontrar los coeficientes adecuados.

Estudiaremos estas dos técnicas en los siguientes ejemplos.

Ejemplo 1.
Demuestra que a7  b7  c7  a 4b3  b4c3  c4a3

Solución.
Vemos que esta desigualdad es homogénea:
 a7   b7   c7  7 a7  b7  c7 
a4 b3  b4 c3  c4 a3  7 a4b3  b4c3  c4a3 
Luego podemos aplicar AM-GM:
4a 7  3b7 4b7  3c 7 4c 7  3a 7 4a 7  3b7
a 7  b7  c 7    
7 7 7 cyc 7
Por otro lado, aplicando la desigualdad AM-GM:
4a 7  3b7 a 7  a 7  a 7  a 7  b7  b7  b7 7 7 7 7 7 7 7 7
  a a a a b b b  a 4b3
7 7
Y por tanto:
4a 7  3b7

cyc 7
  a 4b3  a 4b3  b4c3  c 4 a3
cyc

tal y como queríamos ver.

Ejemplo 2.
a 3 b3 c 3
Demuestra que    abc
bc ac ab

Solución.
Vemos que esta desigualdad es homogénea, luego procedemos a demostrarla mediante
AM-GM:
a 3 b3 c 3 a 4  b 4  c 4
    abc 
bc ac ab abc
a 4  b 4  c 4  abca  b  c   a 2bc  b 2 ac  c 2 ab

Ahora, aplicando AM-GM:


2a 4  b 4  c 4 a 4  a 4  b 4  c 4 4 8 4 4
  a b c  a 2bc
4 4
Y por tanto:
2a 4  b4  c 4 2b4  c 4  a 4 2c 4  a 4  b 4
a b c 
4 4 4
   a 2bc  b2 ac  c 2 ab
4 4 4
2a  b  c
4 4 4
O de forma simplificada:  a 4     a 2bc
cyc cyc 4 cyc
Ejemplo 3.
Suponiendo que a  b  c  1 , demuestra que se cumple
1 1 1 a 3  b3  c 3
   3 2
a b c abc

Solución:
1 1 1 bc  ac  ab a 3  b3  c 3
    3 2  bc  ac  ab  3abc  2a3  b3  c3 
a b c abc abc

Pero ahora vemos que esta desigualdad no es homogénea:


bc  ac  ab  2 bc  ac  ab 
  
3abc  2 a   b   c   3 3abc  2 a3  b3  c3
3 3 3
 
La podemos homogeneizar añadiendo la condición del enunciado a  b  c  1 a la
izquierda:
  
bc  ac  ab  3abc  2 a3  b3  c3  (a  b  c)bc  ac  ab  3abc  2 a3  b3  c3 
(a  b  c)bc  ac  ab   abc  a 2c  a 2b  b 2c  abc  ab 2  bc 2  ac 2  abc 
 3abc  a 2c  a 2b  b 2c  ab 2  bc 2  ac 2

Luego simplificando 3abc se reduce a demostrar la desigualdad:


a 2c  a 2b  b2c  ab2  bc 2  ac 2  2 a3  b3  c3 
a 2c  a 2b  b2c  ab2  bc 2  ac 2   a 2b1c0   a 2b
sym sym

 
2 a3  b3  c3   a3b0c 0   a3
sym sym

Es decir, queremos demostrar:


a  a b
sym
3

sym
2

Esto nos lo garantiza H puesto que 3,0,0  2,1,0

Y se puede demostrar mediante AM-GM de la siguiente manera:


2a3  b3 a3  a3  b3 3 3 3 3
  a a b  a 2b
3 3

2a 3  b3 2b3  c3 2c3  a3 2a 3  c3 2b3  a3 2c3  b3


 
2 a 3  b3  c 3 
3

3

3

3

3

3

 a 2b  b 2 c  c 2 a  a 2 c  b 2 a  c 2b

O escrita de forma simplificada mediante sumatorios:


2a 3  b 3
sym
a 3
 
sym 3
  a 2b
sym
3 El cuadro general de las desigualdades entre medias.
La desigualdad AM-GM es un caso particular del cuadro general de desigualdades
"entre las medias".

3.1 Con dos variables:


ab a 2  b2
min a, b   maxa, b
2ab
 ab  
ab 2 2

3.2 En general:
Si a1 ,...an son números no negativos:

a1  ...  an a12  ...  an2


min ai    max ai 
n
 n a1 ... an  
1 1 n n
 ... 
a1 an
HM GM AM QM

y se produce una igualdad si y solo si a1  ...  an .

Nota: Las siglas se toman del inglés: HM: Harmonic-Mean, GM: Geometric Mean, AM: Arithmetic
Mean, QM: Quadratic Mean.

3.3
Dados dos números reales positivos x, y , se cumple x  y  2( x  y)
y la igualdad solo se cumple cuando x  y .

3.4
Determina el valor máximo de
x  144  722  x

3.5 Problema resuelto.


Variaciones de la desigualdad AM-HM:
1 1 n2
a) a1  ...  an   ...    n 2
1 1
b)  ...  
 a1 an  a1 an a1  ...  an

y se cumple la igualdad si y solo si a1  ...  an

En efecto, partimos de la desigualdad AM-HM:


a1  ...  an 1 1
 a1  ...  an   ...    n 2
n

n 1 1  a1 an 
 ... 
a1 an
3.6 Problema resuelto. Desigualdad de Nesbitt (Inglaterra, 1903):

a b c 3
  
bc ac ab 2

Solución. Sea
a b c a b c bc ac ab
f (a, b, c)          3
bc ac ab bc ac ab bc ac ab
abc bac cab  1 1 
 3  a  b  c 
1
     3
bc ac ab ab bc ac


1
a  a  b  b  c  c  1  1  1   3 
2 ab bc ac


1
a  b   b  c   a  c  1  1  1   3 
2 ab bc ac


1
x  y  z  1  1  1   3 con x  a  b, y  b  c, z  a  c
2 x y z

Primera demostración. Vemos que esta función se adapta perfectamente a la variación


de la desigualdad AM-HM anterior:

f (a, b, c) 
1
x  y  z  1  1  1   3  1 32  3  3
 
2 x y z 2 2

Segunda demostración. Aplicando la desigualdad demostrada anteriormente:

1 1 1
2 f (a, b, c)  x  y  z      6           6 
x x x y y y z z z
x y z x y z x y z x y z
x y x z y z
1    1  1 6 1 2  2 1 2 1 6  3 
y x z x z y
  
2 2 2

3
 f (a, b, c) 
2

Tercera demostración. Aplicando la desigualdad AM-GM:


x yz 3
 xyz  x y z 33 xyz
3
La aplicamos a los dos paréntesis:
1 1 1 111 1 1
  33 33 3
x y z xyz xyz 3 xyz

f (a, b, c) 
1
x  y  z  1  1  1   3  1 3 3 x y z 3 3 1  9  3  3
2 x y z 2 xyz 2 2
4 La desigualdad Cauchy-Schwarz.
4.1 Desigualdad Cauchy-Schwarz.
Para cualquier conjunto de números reales a1 , a2 ,..., an y b1 , b2 ,..., bn ,

a1b1  a2b2  ...  anbn 2  a12  a22  ...  an2 b12  b22  ...  bn2 
a1 a2 a
y la igualdad solo pasa cuando las n-tuplas son proporcionales:   ...  n
b1 b2 bn

Ejemplo resuelto.
Demostrar que, para cualquier número natural n  2 , se cumple que

nn  1
1  2 2  3 3  ...  n n  6n  3
6

Solución: Aplicando la Desigualdad Cauchy-Schwarz tenemos:



S 2  1  1  2 2  3 3  ...  n n  2

 
     
 12  2 2  32  ...  n 2  1  2  3  ... 
2 2 2
 n   
2

 n(n  1)(2n  1)  n(n  1)  n (n  1) (2n  1)


2 2
  
 6  2  12
Luego
n 2 (n  1) 2 (2n  1) 2n  1 n(n  1) 2n  1 n(n  1) 6n  3
S  n(n  1)   
12 12 2 3 2 32
n(n  1)
 6n  3
6

Y vemos que esta desigualdad solo se puede dar en sentido estricto, pues la igualdad se

cumple solo cuando las n-tuplas  1, 2 , 3, ..., n  y 1 , 2 , 3 ,..., n son 
1 2
proporcionales, pero los dos primeros elementos ya no lo son: 1   es absurdo.
1 2

Versión vectorial de la desigualdad Cauchy-Schwarz.


Podemos reescribir la desigualdad anterior en el contexto de la geometría analítica y el
producto escalar de vectores:
 
Sean los vectores de IRn a  a1 , a2 ,..., an  y b  b1, b2 ,..., bn  . Entonces tenemos
a1b1  a2b2  ...  anbn 2  a12  a22  ...  an2 b12  b22  ...  bn2  
a  b   a
 2 2    
b  a b  a b
2

En donde las barras a la derecha de la desigualdad denotan módulo de un vector y a la


izquierda denotan valor absoluto.
Dicho de otra manera, el producto escalar de dos vectores no supera nunca el producto
de sus módulos, y la igualdad se cumple si y solo si los vectores son proporcionales.
4.2 Corolario.
a1  a2  ...  an 2  a 2  a 2  ...  a 2
1 2 n
n

Demostración. Basta tomar b1  b2  ...  bn  1 .

4.3 Desigualdad Cauchy-Schwarz "en la forma de Engel".


Para cualquier conjunto de números reales a1 , a2 ,..., an y para cualquier conjunto
b1 , b2 ,..., bn de números positivos,

 2
a22 a2 
a1  a2  ...  an 2   a1   ...  n  b1  b2  ...  bn 
 b1 b2 bn 

Demostración.
ai
Basta aplicar la desigualdad Cauchy-Schwarz con a'i  y b'i  bi .
bi

4.4 Corolario a la Desigualdad Cauchy-Schwarz "en la forma de Engel".


Para cualquier conjunto b1 , b2 ,..., bn de números reales positivos,

1 1 1
n 2  b1  b2  ...  bn    ...  
 b1 b2 bn 

y la igualdad acontece si y solo si b1  b2  ...  bn .

Demostración.
Basta tomar a1  a2  ...  an  1
Alternativamente, se puede demostrar mediante la desigualdad AM-GM:
b1  b2  ...  bn  n n b1b2 ...bn 
 1 1 1
1 1 1   b1  b2  ...  bn  b  b  ...  b   n
2
1 1 1
  ...   n n ...   1 2 n 
b1 b2 bn b1 b2 bn 

Nota histórica.
Bunyakovskii (1804-1889) publicó esta desigualdad en una monografía sobre
desigualdades entre integrales en 1859, veinticinco años antes que Schwarz (1843-
1921), pero es más conocida como desigualdad de Cauchy-Schwarz.
4.5
Demostrar que la desigualdad AM-QM es un caso particular de la desigualdad Cauchy-
Schwarz.

4.6
Sean x, y, z números reales positivos tales que x  y  z  1. Determina el valor mínimo
1 4 9
de   .
x y z

4.7
Encontrar el máximo de la función f ( x)  a sin x  b cos x , con a, b  0 , y 0  x   / 2 .

4.8
Sean a, b, c, d , e números reales tales que a  b  c  d  e  8 y
a 2  b2  c2  d 2  e2  16

Hallar el valor máximo de e.

4.9
Demostrar que si a  b  c  3 , entonces a 2  b2  c 2  3

4.10
 
Demostrar que si a, b, c  0 , entonces a 2b  b2c  c2a ab2  bc 2  ca 2  9a2b2c 2
4.11
1 1 4 16 64
Demuestra que si a, b, c, d  0 , entonces    
a b c d abcd

4.12 M
Sean a, b, c números reales positivos. Demostrar que si
a2 b2 c2
  1
a 2  1 b2  1 c 2  1

3
Entonces ab  bc  ca  .
2

4.13 F
Demuestra el siguiente corolario a la desigualdad Cauchy-Schwarz:
a12  b12  a22  b22  a1  a2 2  b1  b2 2

y acontece la igualdad si y solo si las parejas (a1 , a2 ) y (b1 , b2 ) son proporcionales.

Que se puede generalizar por inducción al caso de n números:


a12  b12  a22  b22  ...  an2  bn2  a1  ...  an 2  b1  ...  bn 2

y acontece la igualdad si y solo si las tuplas (a1 ,..., an ) y (b1 ,..., bn ) son proporcionales.
4.14 MD
Sean a, b, c  0 . Demostrar que
a 2  bc b 2  ca c 2  ab
  0
2a 2  b 2  c 2 2b 2  c 2  a 2 2c 2  a 2  b 2
Pham Kim Hung

4.15 F
1 1 1
Supongamos que x, y, z  1 y    2 . Demostrar que
x y z

x  y  z  x 1  y 1  z 1

IRAN MO 1998

4.16 F
Sean x, y, z reales positivos tales que x  y  z  3 . Halla el valor máximo
alcanzado por
x  2 y  2  3z  6

¿Para qué valores de x, y, z se alcanza dicho máximo?

OME 2015 Primera fase, segunda sesión #6


Notas biográficas.
Augustin Louis Cauchy (París, 21 de agosto de 1789 – Sceaux, 23 de mayo de 1857) fue
un matemático e ingeniero francés. Estudió Ingeniería en la Escuela Politécnica de
París, donde trabajó como profesor de mecánica (1816), y posteriormente, fue ascendido
a miembro de la Academia Francesa de las Ciencias.

Su contribución matemática es especialmente notable en la rama del análisis


matemático. En 1814 publicó la memoria de la integral definida, que llegó a ser la base
de la teoría de las funciones complejas. Precisó los conceptos de función, límite y
continuidad de manera muy similar a como los conocemos en la actualidad, tomando el
concepto de límite como punto de partida del análisis y la noción de correspondencia
como idea de función.

También hizo aportaciones a la probabilidad, las ecuaciones diferenciales y la física


matemática. Investigó la convergencia y divergencia de las series infinitas y demostró el
Teorema del número poligonal de Fermat, al que se habían dedicado ilustres
matemáticos contemporáneos sin éxito.
Poco antes de morir pronunció la frase: “No me imagino una vida más plena que una
vida dedicada a la matemática”, mostrando arrepentimiento por el que consideraba su
único error en la vida: no haber dedicado más tiempo a las matemáticas.

Karl Hermann Amandus Schwarz (Polonia, 25 de enero de 1843 – Berlín, 30 de


noviembre de 1921), fue un matemático alemán conocido por su trabajo en análisis
matemático. Estudió Química en Berlín aunque fue persuadido por Kummer y
Weierstrass para que se centrara en las matemáticas.

Desde 1875 trabajó en la Universidad de Gotinga, donde trató temas como la teoría de
funciones, geometría diferencial y cálculo de variaciones. Su trabajo Búsqueda de una
superficie mínima, acabado en 1867, fue incluido en su Colección de artículos
matemáticos (1890).
En 1892 se convirtió en miembro de la Academia de las Ciencias de Berlín y fue
profesor de la Universidad de esta ciudad. Entre sus alumnos más importantes se
encuentran Lipót Fejér, Paul Koebe y Leon Lichtenstein.

Viktor Yakovlevich Bunyakovsky (Ucrania, 16 de diciembre San Petersburgo, 12 de


diciembre de 1889) fue un matemático ruso, miembro y posterior vicepresidente de la
Academia de Ciencias de San Petersburgo.

Estudió Matemáticas en la Universidad de la Sorbona, donde se doctoró bajo la


supervisión de Cauchy. Dedicó gran parte de su vida a la enseñanza e investigación.
Contribuyó de manera significativa en la teoría de números y en la teoría de la
probabilidad, aunque también hizo aportaciones en ciencias como la física, mecánica y
finanzas. Se le atribuyen más de 150 publicaciones sobre trabajos de investigación.
5 El principio de reordenación. La desigualdad de Chebyshev.
El principio de reordenación de dos elementos.
5.1 MF
Demuestra que, si a  b y x  y , entonces ax  by  ay  bx .

A esta desigualdad la llamaremos "Principio de Reordenación de dos elementos".

5.2 MF
Demuestra que, si x, y  0 ,
x2 y2
  x y
y x

5.3 F
Sean a, b, c, d números reales tales que a  d  b  c . Demostrar que

(a  b)(c  d )  (a  c)(b  d )  (d  a)(b  c)  0

Czech and Slovak Republics, 2004

El principio de reordenación general.


El Principio de reordenación de dos elementos introducido en el apartado anterior se
puede extender a cualquier n-tupla de números:

Dadas dos secuencias ordenadas de números: a1  a2  ...  an y b1  b2  ...  bn , la


suma a1b1  a2b2  ...  anbn es maximal, y la suma anb1  an 1b2  ...  a1bn es minimal.
Esto quiere decir que la suma con cualquier otra permutación a'1 , a'2 ,..., a'n  se
encontrará entre estas dos:

anb1  an 1b2  ...  a1bn  a'1 b1  a'2 b2  ...  a'n bn  a1b1  a2b2  ...  anbn

Además, se cumple
anb1  an 1b2  ...  a1bn  a'1 b1  a'2 b2  ...  a'n bn  a'1 , a'2 ,..., a'n   a1, a2 ,..., an 

y se cumple
a'1 b1  a'2 b2  ...  a'n bn  a1b1  a2b2  ...  anbn  a'1 , a'2 ,..., a'n   an , an 1,..., a1 

Nota:
Para simplificar la escritura, en las primeras soluciones de este tema se utiliza la
siguiente notación:
a1 a2 a3 
b b b   a1b1  a2b2  a3b3
 1 2 3
5.4 D
Demostrar la desigualdad AM-GM aplicando el Principio de Reordenación.

5.5
Demostrar que a 2  b2  c2  ab  bc  ac

5.6
Demostrar que a3  b3  c3  ac2  ba2  cb 2

Nota: Este problema ya fue propuesto en 2.20. Se propone ahora resolverlo mediante reordenación.

5.7
Demostrar la desigualdad de Nesbitt introducida en 3.6 aplicando la técnica de este
apartado.

5.8
Demostrar que, dados a1 ,..., an números reales positivos, y s  a1  ...  an , se cumple:
a1 a a n
 2  ...  n 
s  a1 s  a2 s  an n  1

5.9
Demostrar que si a, b, c  0 , entonces
a 2  bc b 2  ac c 2  ab
   abc
bc ac ab

Nota:
Los problemas 10.12 y 10.15 de desigualdades con los lados del triángulo están
resueltos mediante el principio de reordenación.
La desigualdad de Chebyshev.

Versión directa de la desigualdad de Chebyshev.


Sean dos secuencias ai  y bi  ordenadas de la misma forma (es decir, ambas
crecientes o ambas decrecientes). Entonces, aplicando el principio de la reordenación
que acabamos de ver, tenemos:

a1b1  ...  anbn  a1b1  a2b2  ...  anbn


a1b1  ...  anbn  a1b2  a2b3  ...  anb1
a1b1  ...  anbn  a1b3  a2b4  ...  anb2
...........................................................
a1b1  ...  anbn  a1bn  a2b1  ...  anbn 1

Sumando todas estas desigualdades obtenemos:

n a1b1  ...  anbn   a1  ...  an b1  ...  bn 

Versión inversa de la desigualdad de Chebyshev.


Si las secuencias ai  y bi  están ordenados de forma inversa, es decir, una es creciente
y la otra decreciente, tenemos la desigualdad contraria:

a1  ...  an b1  ...  bn   n a1b1  ...  anbn 


5.10 Problema resuelto.
Demostrar que, si a, b, c  0 ,
a b c
  1
a 2  8bc b 2  8ca c 2  8ab
IMO 2001 #2
Solución:

Aplicando 2 xy  x 2  y 2 ,
a2  8bc  a 2  2bc  6bc  a2  b2  c2  6bc  S  6bc

donde S  a 2  b2  c2 . Luego
a b c a b c
    
a  8bc
2
b  8ca
2
c  8ab
2
S  6bc S  6ac S  6ab

Vemos que las secuencias


 1 1 1 
(a, b, c) y  , , 
 S  6bc S  6ac S  6ab 

están ordenadas de la misma forma, y por tanto podemos aplicar la desigualdad de


Chebyshev:
 
 a  b  c 
a b c 1 1 1 1
    
S  6bc S  6ac S  6ab 3  S  6bc S  6ac S  6ab 

Ahora aplicamos la desigualdad AM-HM (ver 3.5b):


1 1 1 9
  
S  6bc S  6ac S  6ab S  6bc  S  6ac  S  6ab

Y aplicamos la desigualdad QM-AM:


3 3
  (*)
S  6bc  S  6ac  S  6ab S  6bc   S  6ac   S  6ab
Finalmente observamos que
S  6bc  S  6ac  S  6ab  3S  2(ab  bc  ac)  3a  b  c2
Y por tanto
3 1
(*)  
3a  b  c 
2
a  b  c 
Con lo que llegamos a:
 a  b  c 
a b c 1
  1
S  6bc S  6ac S  6ab abc

Tal y como queríamos ver. La igualdad se cumple cuando a  b  c  1 .

Nota: Este mismo problema se volverá a resolver en 6.4 mediante otra técnica.

Fuente: https://www.artofproblemsolving.com/Wiki/index.php/2001_IMO_Problems/Problem_2

5.11 F
Sean a, b, c, d números reales positivos tales que a 2  b2  c2  d 2  4 . Demostrar que
a2 b2 c2 d2 4
   
bcd cd a d ab abc 3
Nota biográfica.
Pafnuty Lvovich Chebyschov (Okatovo, 16 de mayo de 1821 – San Petesburgo, 8 de
diciembre de 1894) es conocido principalmente por su trabajo en las áreas de
probabilidad y estadística.

Comenzó sus estudios universitarios de Matemáticas en 1837 y cuatro años después, los
terminó destacando por su nivel. En 1846 defendió su tesis "Un intento de análisis
elemental de la teoría probabilística". Fue profesor de matemáticas en San Petesburgo y
fundó una importante escuela de matemáticos en esta ciudad. Más adelante se convirtió
en miembro de la Academia Imperial de Ciencias y posteriormente, fue elegido
miembro honorario de la Sociedad Matemática de San Petesburgo (1893).
Según mencionó él mismo, fue su profesora de música de la infancia la que “llevó su
mente a la exactitud y el análisis”. Se dedicó al estudio de la teoría de los números y el
cálculo de probabilidades. Entre sus aportaciones más notables destacan la
generalización de la Ley de los grandes números, el Teorema del límite central y las
desigualdades que llevan su nombre.
6 La desigualdad de Jensen.
Fuente fundamental de este capítulo: Secrets in Inequalities (volume 1) (Pham Kim Hung), páginas 67 en adelante.

6.1 Definición. Función convexa.


Diremos que una función f (x) es convexa en a, b cuando, para cualquier par x  y
en a, b se cumpla
t f  x   (1  t ) f  y   f t x  (1  t ) y 

es decir, cuando la gráfica de la función está siempre por debajo del segmento que une
dos de sus puntos.

6.2 Teorema.
Si f ' ' ( x)  0 en a, b , entonces la función es convexa en a, b .

Demostración. Tomamos valores a  x  y  b y 0  t  1 . Fijando los valores y, t ,


consideremos la función
g ( x)  tf ( x)  (1  t ) f ( y)  f tx  (1  t ) y  .

Entonces
g ' ( x)  tf ' ( x)  t f ' tx  (1  t ) y   t  f ' ( x)  f ' tx  (1  t ) y 

Por hipótesis, f ' ' ( x)  0 ,y por tanto f ' ( x) es creciente. Luego:


x  tx  (1  t ) y
 f ' ( x)  f ' tx  (1  t ) y   f ' ( x)  f ' tx  (1  t ) y   0
 t  f ' ( x)  f ' tx  (1  t ) y   0  g ' ( x)  0

Es decir, la función g (x) es decreciente, y por tanto, puesto que en todo momento
estamos suponiendo x  y , tenemos que g ( x)  g ( y)  0 .

Finalmente:
0  g ( x)  tf ( x)  (1  t ) f ( y)  f tx  (1  t ) y  
tf ( x)  (1  t ) f ( y)  f tx  (1  t ) y 

tal y como queríamos ver.


6.3 Teorema. Desigualdad de Jensen.
Si f es una función convexa en a, b  IR , entonces, para todo x1, x2 ,..., xn  a, b , y
para todo a1 , a2 , ..., an  0 cumpliendo a1  a2  ...  an  1 , se cumple
a1 f ( x1 )  a2 f ( x2 )  ...  an f ( xn )  f a1 x1  a2 x2  ...  an xn 

En particular, tomando a1  a2  ...  an  1 / n , tenemos el siguiente corolario:

Si f es una función convexa en a, b  IR , entonces, para todo x1, x2 ,..., xn  a, b ,
 x  x  ...  xn 
f ( x1 )  f ( x2 )  ...  f ( xn )  n f  1 2 
 n 

Nota: Si la función es cóncava se verifican las desigualdades contrarias.

6.4 Problema resuelto.


Demostrar que, si a, b, c  0 ,
a b c
  1
a  8bc
2
b  8ca
2
c  8ab
2

IMO 2001 #2

Solución:

Primera versión.

Puesto que la desigualdad es homogénea, podemos suponer a  b  c  1 (ver Tema 7).


1
La función f ( x)  es convexa, luego podemos aplicar la desigualdad de Jersen:
x
1 1 1 1
a b c  (*)
a  8bc
2
b  8ca
2
c  8ab
2
a(a  8bc)  b(b  8ca )  c(c 2  8ab)
2 2

Y ahora tenemos en cuenta que a  b  c  1 , y por tanto



1  a  b  c   a 3  b3  c3  6abc  3 a 2b  a 2c  b 2 a  b 2c  c 2 a  c 2b 
3

 a  b  c  6abc  3(a  b)(b  c)(a  c)  2abc  
3 3 3

a 3  b3  c3  3(a  b)(b  c)(a  c)  a 3  b3  c3  24abc


En donde hemos tenido en cuenta (a  b)(b  c)(a  c)  8abc (ver problema 2.3)

Finalmente: a3  b3  c3  24abc  a  b  c   (*) 


1 1
 1
3

( a  b  c) 3 abc

Segunda versión.
De nuevo, puesto que la desigualdad es homogénea, podemos suponer a  b  c  1 (ver
Tema 7).
a b c a b c
     
a  8bc
2
b  8ca
2
c  8ab
2
a 8
2 abc
b 
2 abc
c 8
2 abc
a b c
a2 b2 c2 a2 b2 c2
     
abc abc abc abc abc abc
a2  8 b2  c2  8 a2  8 b2  8 c2  8
a b c a b c
a3 b3 c3
  3  3
a 3  8abc b  8abc c  8abc

x3
La función f ( x)  es cóncava, y por tanto podemos aplicar la desigualdad
x3  8abc
de Jersen:
1 1 1 
f (a)  f (b)  f (c)  3 f  a  b  c 
3 3 3 

Aplicando la desigualdad AM-GM, y teniendo en cuenta que la función f (x) es


estrictamente creciente:
1 1 1 1 1 1  1
a  b  c  3 abc  3 f  a  b  c   3 f 3 abc  3  1  
3 3 3 3 3 3  3

Fuente: https://www.artofproblemsolving.com/Wiki/index.php/2001_IMO_Problems/Problem_2

6.5
Dados a, b, c números reales no negativos tales que a  b  c  1 , demostrar que
1
a b b c c a 
3

6.6
Demuestra la desigualdad AM-GM como caso particular de la desigualdad de Jensen.

6.7
Si a, b, c  0 y a  b  c  1 , determina el mínimo de
10 10 10
 1  1  1
 a    b    c  
 a  b  c

6.8
Demostrar que, en todo triángulo ABC ,
3 3
sin A  sin B  sin C 
2
es decir, el mínimo perímetro de un triángulo inscrito en una circunferencia fija se
obtiene con el triángulo equilátero.
6.9
Demostrar que, en todo triángulo ABC ,
3 3  tan A  tan B  tan C

6.10
Demostrar que, en todo triángulo ABC ,
A B C
3  tan  tan  tan
2 2 2

6.11 D
Suponiendo que a1 , a2 , ... , an   1 / 2 , 1  , demostrar que
a1 a2 ... an (1  a1 )(1  a2 )...(1  an )

a1  a2  ...  an  n  a1  a2  ...  an n
n

Las tres proposiciones siguientes nos permiten ahorrarnos verificar la convexidad de la función:

6.12 Proposición.
Si una función real f : a, b  IR satisface la condición
x y
x, y  a, b  f ( x)  f ( y)  2 f  
 2 

Entonces, para todo x1, x2 ,..., xn  a, b , se cumple


 x  x  ...  xn 
f ( x1 )  f ( x2 )  ...  f ( xn )  n f  1 2 
 n 

6.13 Proposición.
Si una función real f : a, b  IR satisface la condición
x, y  a, b  f ( x)  f ( y)  2 f  xy 
Entonces, para todo x1, x2 ,..., xn  a, b , se cumple
f ( x1 )  f ( x2 )  ...  f ( xn )  n f 
n x1 x2 ... xn 
Observación.
La proposición anterior se puede generalizar a cualquier tipo de media: Aritmética, geométrica,
armónica...

6.14 Proposición.
Si f es una función definida en a, b  IR , entonces, para todo x1, x2 ,..., xn  a, b , y
para todo a1 , a2 , ..., an  0 cumpliendo a1  a2  ...  an  1 , se cumple
a1 f ( x1 )  a2 f ( x2 )  ...  an f ( xn )  f a1 x1  a2 x2  ...  an xn 

si y solo si se cumple para el caso n  2 .


6.15 Problema resuelto.
Supongamos que x1 , x2 ,..., xn  1 . Demostrar que
1 1 1 n
  ...  
1  x1 1  x2 1  xn 1  n x1 x2 ...xn
IMO Shortlist

Solución: Aplicando 6.13, es suficiente demostrar que, para todo x, y  1 ,


1 1 2
 
1 x 1 y 1 x y

Que es equivalente a demostrar


1 1 2
 
1 x 1 y
2 2
1 x y
1 1 2 2  y2  x2 2
    
1 x 1 y
2 2
1 x y 1 x 1 y
2

2
1 x y  
2  y 2
  
 x 1  x y   2 1  x 1  y 
2 2 2

x  y  2  x y  y x  2 xy  2  2 x 2  2 y 2  2 x 2 y 2 
2 2 3 3

x 3 y  y 3 x  2 xy  x 2  y 2  2 x 2 y 2  x 2  y 2  2 x 2 y 2  x 3 y  y 3 x  2 xy  0 
( x  y ) 2 (1  xy )  0

Lo cual es cierto, pues x, y  1  xy  1  1  xy  0  ( x  y) 2 (1  xy )  0

Nota biográfica:
Johan Ludwig William Valdemar Jensen (Nakskov, 8 de mayo de 1859; Copenhague, 5
de marzo de 1925), más conocido como Johan Jensen, fue un matemático e ingeniero
danés. Con 17 años fue inscrito en el Colegio de Tecnología de Copenhague, ciudad en
la que años más tarde se convirtió en respetado ingeniero de la Compañía Telefónica,
llegando a ocupar el puesto de Jefe de la Oficina Técnica de Investigación (1890).
Aunque estudió Matemáticas en la Universidad, su conocimiento a nivel avanzado de
esta materia lo logró de manera autodidacta. Todos sus trabajos matemáticos los llevó a
cabo en su tiempo libre. Publicó un trabajo de investigación y presidió la Sociedad
Matemática Danesa de 1892 a 1903.

Ha pasado a la historia por ser la primera persona que se interesó por un fenómeno
característico de las funciones continuas no lineales (cóncavas o convexas), conocido
como Desigualdad de Jensen. Esta propiedad aparece en múltiples contextos y tiene
diversas aplicaciones en ámbitos como el financiero o el sanitario.
El truco de la recta tangente (“Tangent Line Trick”).

6.16 Ejemplo resuelto.


Determina el valor mínimo de la expresión
a b c d
 3  3  3
b 4 c 4 d 4 a 4
3

donde a, b, c, d son reales no negativos tales que a  b  c  d  4 .

USAMO 2017 #6

Solución:
Podemos escribir:
a b c d
 3  3  3  a f (b)  b f (c)  c f (d )  d f (a)
b 4 c 4 d 4 a 4
3

, x  0,4
1
con f ( x) 
x 4
3

Vamos a demostrar que la solución es 2 / 3 . Observamos que no podemos aplicar la


“Desigualdad de Jensen” porque la función f (x) no es convexa en este intervalo:

Lo que sí podemos es encontrar otra función g ( x)  f ( x) en este intervalo. De hecho,


1 x
podemos encontrar la mejor función lineal g ( x)  f ( x) , que es g ( x)  
4 12

Luego:
a b c d
 3  3  3  a f (b)  b f (c)  c f (d )  d f (a) 
b 4 c 4 d 4 a 4
3

 a g (b)  b g (c)  c g (d )  d g (a) 


1 b  1 c  1 d  1 a 
 a    b    c    d    
 4 12   4 12   4 12   4 12 
a ba b cb c dc d da a  b  c  d ab  bc  cd  da
          
4 12 4 12 4 12 4 12 4 12
4 ab  bc  cd  da ab  bc  cd  da
   1 (*)
4 12 12

Por otro lado vemos que, aplicando la desigualdad AM-GM, tenemos

ab  bc  cd  da  (a  c)(b  d ) 
a  b  c  d  42
2
 4
4 4

Y por tanto:
4 2
(*)  1  
12 3

Tal y como queríamos ver.

Queda por demostrar que f ( x)  g ( x) . En efecto:


1 x 3 x
 12  x3  43  x  
1
f ( x)  g ( x)  3   
x  4 4 12 12
 0  x  43  x   12   x( x  2)2 ( x  1)
3

Lo cual es obviamente cierto para todo x  0 .

Observación: Vemos que se cumple g ( x)  f ( x) en los puntos  0 ,1 / 4  y  2 ,1 / 12  ,


y por tanto podemos deducir que la igualdad se alcanza cuando dos de los valores son
iguales a 2 y los otros dos son iguales a 0, por ejemplo:

a  b  2 a b c d 2 2 2 1 2
 3  3  3  3     
c  d  0 b  4 c  4 d  4 a  4 8  4 0  4 12 2 3

Curiosamente, tomando la hipótesis “natural” a  b  c  d  1 hubiéramos obtenido


4/5, que no es la solución correcta.

Fuentes:
Solución oficial.
USAMO 2017 Solution Notes (Evan Chen).
https://dgrozev.wordpress.com/2020/04/16/а-weaker-jensens-inequality-an-olympiad-approach/

6.17 F
Sea un entero n  3 , y x1, x2 , ..., xn números reales no negativos tales que
x1  x2  ...  xn  2 . Determina el valor mínimo de
x x x
A( x1, x2 ,..., xn )  2 1  2 2  ...  2 n
x2  1 x3  1 x1  1

CGMO 2007 #3
7 Desigualdades simétricas. Normalización y homogeneización.

7.1 Definición. Desigualdades simétricas.


Una desigualdad simétrica es aquella que se puede expresar como f  a1, a2 ,..., a1   0
 
Cumpliendo f ai1 , ai2 ,..., ain  f  a1 , a2 ,..., a1  para cualquier permutación i1 , i2 ,..., in 
de 1,2 , ..., n  . Un ejemplo clásico de desigualdad simétrica es la Desigualdad de Schur:

7.2 Problema resuelto. Desigualdad de Schur.


Suponiendo a, b, c  0 , entonces
a3  b3  c3  3abc  ab(a  b)  bc(b  c)  ca(c  a)

Solución.
a 3  b3  c3  3abc  ab(a  b)  bc(b  c)  ca (c  a) 
a 3  b3  c3  3abc  ab(a  b)  bc(b  c)  ca (c  a)  0 
a(a  b)(a  c)  b(b  c)(b  a)  c(c  a)(c  b)  0

Aprovechando que la desigualdad es simétrica, podemos suponer que a  b  c .


Sean x  a  b , y  b  c . Entonces:
x  y  a bbc  a c  a  x  y c, b  y  c
y la desigualdad queda:
0  ( x  y  c) x( x  y)  ( y  c) y( x)  c(( x  y))( y) 
 x( x  y  c)( x  y)  xy ( y  c)  cy ( x  y)  c( x 2  xy  y 2 )  x 2 ( x  2 y)

Lo cual es cierto porque c, y, c  0 . La igualdad acontece cuando x  y  0 y x  c  0 ,


es decir, cuando a  b  c o cuando a  b , c  0 (o cualquiera de sus permutaciones).

7.3 Observación.
Esta desigualdad es equivalente a
(a  b  c)(b  c  a)(c  a  b)  abc con a, b, c  0 ,
que fue demostrada independientemente en el problema 2.12.

7.4 Desigualdad de Schur generalizada.


Si a, b, c  0 y tomando cualquier constante k  0 , se cumple:
a k (a  b)(a  c)  bk (b  a)(b  c)  ck (c  a)(c  b)  0

7.5 Observación.
Se puede demostrar que la desigualdad de Schur es cierta también para k  0 . Y se
puede demostrar que si k es par, la desigualdad es cierta para cualquier terna a, b, c no
necesariamente positivos.

Nota biográfica.
Issai Schur (Bielorrusia, 10 de enero de 1875 Israel, 10 de enero de 1941), estudiante de
Ferdinand Georg Frobenius, fue conocido principalmente por su trabajo en
representaciones de grupos, aunque también trabajó en combinatoria y física teórica.
Homogeneización.

7.6 Definición. Inecuaciones homogéneas.


Decimos que una función f (a, b, c,...) es homogénea de grado n si, para todo k  0 ,
f (ka, kb, kc,...)  k n f (a, b, c,...)

En particular, diremos que es homogénea de grado 0 si f (ka, kb, kc,...)  f (a, b, c,...)

a
Por ejemplo, la función f (a, b, c)  con a, b, c,  0 es homogénea de grado 0
a  8bc
2

puesto que
ka ka a
f (ka, kb, kc)     f (a, b, c)
ka2  8kbkc k a 2  8bc a 2  8bc

Llamaremos desigualdad homogénea de grado n a toda desigualdad de la forma


f (a, b, c,...)  0 con f (a, b, c,...) homogénea de grado n.

Las desigualdades homogéneas de grado 1 se pueden "escalar", es decir, podemos


multiplicar sus variables por cualquier valor de k  0 . En efecto:

0  f (a, b, c,...)  k f (a, b, c,...)  0  f (k a, k b, k c,...)

Homogeizar una inecuación es utilizar la condición dada en el enunciado convertirla en


una inecuación homogénea equivalente.

Para poder aplicar la desigualdad AM-GM, debemos homogeinizar la desigualdad.

7.7 F
Si a, b, c  0 y a  b  c  1 , demostrar que
a 2  b2  c 2  1  4ab  bc  ca 

7.8 M
Sean a, b, c números reales positivos tales que abc  1. Demostrar que
a b c
   abc
b c a

7.9 M
Sean a, b, c números reales positivos tales que abc  1. Demostrar que
a  b  c  a 2  b2  c 2
Normalización.

7.10 Definición. Normalización.


Las desigualdades homogéneas se pueden normalizar, es decir, podemos añadir
restricciones que no figuraban en el enunciado.
Por ejemplo, supongamos que queremos demostrar a3  b3  c3  3abc  0 . Aunque no
figura en el enunciado, podemos añadir la condición abc  1 . En efecto, supongamos
que abc  k 3 , y sean a  ka' , b  kb' , c  kc' .

Entonces a' b' c'  1 , y nuestra desigualdad se convierte en a'3 b'3 c'3 3a' b' c'  0 , que es
la misma que la anterior.

7.11 M
Sean a, b, c números positivos. Demostrar que
1 1 1 1
 3 3  3 
a  b  abc b  c  abc c  a  abc abc
3 3 3

7.12
Demostrar que, si a, b, c  0 ,
a b c
  1
a  8bc
2
b  8ca
2
c  8ab
2

IMO 2001 #2

7.13 Observación.
Una lista ampliada de condiciones que se pueden añadir a una desigualdad homogénea
podría ser la siguiente:

a) a  1 b) b  1 c) c  1 d) abc  1 e) ab  bc  ca  1 f) a 2  b2  c 2  1

7.14 F
Sean a, b, c números reales positivos tales que abc  1. Demuestra que
 1  1  1
 a  1   b  1   c  1    1
 b  c  a

IMO 2000 #2
8 Problemas olímpicos y preolímpicos con desigualdades algebraicas.
8.1 F
a) Demuestra que
1 3 5 7 99 1
   ... 
2 4 6 8 100 10

b) Demuestra, por inducción, que


1 3 5 7 2n  1 1
   ...  n 1
2 4 6 8 2n 3n  1

c) Con ayuda del apartado anterior, mejora la desigualdad del apartado (a) demostrando
que
1 3 5 7 99 1
   ... 
2 4 6 8 100 12

8.2 MF
Demostrar que si a, b, c, d  0 , entonces

 
16 a3  b3  c3  d 3  a  b  c  d 
3

8.3 F
Supongamos que a  b  c  1 , siendo a, b, c  0 . Demostrar que

9a  1  9b  1  9c  1  6

8.4 MF
Sean a, b, c números positivos. Demostrar que

abc 1 1 1
 2 2 2
abc a b c

8.5 M
Sean a, b, c números reales positivos tales que abc  1. Prueba la desigualdad siguiente:

2 2 2
 a   b   c  3
      
 1  ab   1  bc   1  ac  4

OME 2009 #5
8.6 MD
Sean a, b, c  0 cumpliendo
a 2  b2  c2  abc  4

demostrar que
0  ab  bc  ca  abc  2

USAMO 2001 #3

8.7 M
Sean a, b, c números reales positivos tales que
a 2  b2  c 2  a  b  c   4
2

Demostrar que
ab  1 bc  1 ca  1
  3
a  b b  c  c  a 2
2 2

USAMO 2011 #1

8.8 D
Dados a, b, c  0 , demostrar que
2a  b  c 2  2b  c  a 2  2c  a  b2  8
2a 2  b  c  2b 2  c  a  2c 2  a  b 
2 2 2

USAMO 2003 #5

8.9 F
Dados a1 , a2 , ..., an enteros positivos diferentes, demostrar que
n n
ak 1

k 1 k
2
 
k 1 k

IMO 1978 #5

8.10 F
Demuestra que
( a  b) 2 a  b ( a  b) 2
  ab 
8a 2 8b

para todo a  b  0 .
8.11 F
Sean a, b y n enteros positivos tales que a  b y ab  1  n2 . Prueba que

a  b  4n  3 .

Indica justificadamente cuando se alcanza la igualdad.

OME 2013 #1

8.12 F
Sean a, b números positivos. Probar que
a 2  b2
a  b  ab 
2

OMEFL 2014 segunda sesión #4

8.13 D
Sean a, b, c tres números reales positivos. Demuestra que
a  b  3c a  3b  c 3a  b  c 15
  
3a  3b  2c 3a  2b  3c 2a  3b  3c 8
OME 2010 segunda sesión #4

8.14 F
Demuestra que
ax  by 2  ax2  by 2
para cualesquiera x, y  IR y cualesquiera a, b  IR con a  b  1 , a, b  0 .
¿En qué casos se da la igualdad?

OME 2015 Primera Fase, primera sesión #1

8.15 F
Dados dos números reales positivos p, q tales que p  q  1, y sabiendo que todo par de
números reales x, y cumple x  y   0 , se pide demostrar
2

x y
a)  xy
2
x2  y 2  x  y 
2

b)  
2  2 
2 2
 1  1 25
c)  p     q   
 p  q 2

OME 1984 #3
8.16 MD
Sean x e y números reales entre 0 y 1. Probar que
x3  xy 2  2 xy  2 x 2 y  x 2  x  y

OME 2014 Primera fase, segunda sesión #5

8.17 MD
Prueba que para cualesquiera números reales a,b tales que 0  a, b  1 , se cumple la
desigualdad siguiente:
ab2  a 2b  (1  a)(1  b)2  (1  a)2 (1  b)  2

OME 2008 #2

8.18 D
a) Demostrar que
x2 y2 z2
  1
( x  1)2 ( y  1) 2 ( z  1) 2

Para todos los números reales x, y, z , todos diferentes de 1 y cumpliendo xyz  1 .

b) Demostrar que la igualdad acontece para infinitas ternas de números racionales


x, y, z , todos diferentes de 1 y cumpliendo xyz  1 .

IMO 2008 #2

Nota: Solo el apartado (a) es un problema de desigualdades. El apartado (b) es un


problema propio de Teoría de Números.

8.19 D

Determina el valor máximo de la expresión


2 t

 3t t
con t real.
4t

1 1 1 1 1
(A) (B) (C) (D) (E)
16 15 12 10 9
AMC 12B 2020 #22

8.20 D
Sean a, b, c, d números reales tales que a  b  c  d  0 y a  b  c  d  1 .
Demuestra que
a  2b  3c  4d  a abbcc d d  1
IMO 2020 #2
8.21 D
Sean a, b, c, d números reales tales que
a  b  c  d  0 y a 2  b2  c2  d 2  12

Halla el valor mínimo y el valor máximo que puede tomar el producto abcd , y
determina para qué valores de a, b, c, d se consiguen ese mínimo y ese máximo.

OME 2021 #4
9 Desigualdades trigonométricas.
9.1 F
Demuestra que


1 
a 
 1 
b 
  1  2ab   2

 sin x   cos x 


para cualquier a, b, x con a, b  0 y 0  x 
2

9.2 F
Determina el valor mínimo de
9 x 2 sin 2 x  4
f ( x) 
x sin x

para 0  x  
AIME 1983 #9

9.3 MF
Determina el valor mínimo de

sin 3 x cos 3 x 
f ( x)   0 x
cos x sin x 2

9.4 F
Demuestra que

1 
a 
 1 
b 
  1  2ab  
2

 sin x   cos x 


para todo a, b, x tales que a, b  0 y 0  x  .
2
10 Desigualdades con los lados del triángulo.
10.1 Teorema. La Desigualdad Triangular.
Cuando trabajamos con triángulos debemos tener muy en cuenta que los números
a , b , c  0 deben cumplir, además, la Desigualdad Triangular (ver GA/3.7.7):

La suma de dos lados siempre es mayor que el lado restante

a) a  b  c , b  c  a , c  a  b

O equivalentemente:

b) a  b  c , b  a  c , c  a  b

c) (a  b  c)(b  c  a)(c  a  b)  0

Demostración:

a  b)
a  b  c   a  b  c  a  0  b  c  a  2a 
0  b  c  a  0  b  a  c  c  b  a

b  c  a  2a  b  c  a  b  c  a  b  a  b
Las otras dos condiciones se demuestran de forma análoga.

a  c )La expresión (a  b  c)(b  c  a)(c  a  b) es el producto de tres factores


positivos, por lo tanto es un número positivo.

c  a )Por hipótesis, (a  b  c)(b  c  a)(c  a  b)  0 , y por tanto, o bien los tres


factores son positivos, es decir, se cumplen las condiciones del apartado a, o bien dos de
ellos son negativos y el restante positivo. Pero esto no puede suceder. Por ejemplo, si
a  b  c  0
  0  a  b  c  b  c  a  2b  b  0
b  c  a  0
Lo cual no puede suceder pues estamos trabajando con valores positivos.
10.2 Teorema. La Transformación de Ravi.
Las condiciones sobre los lados a , b , c  0 que acabamos de ver pueden ser sustituidas
por la existencia de tres valores x, y, z  0 tales que

a x y, b y z, c zx ,

Demostración.  Sea s  (a  b  c) / 2 . Basta tomar


x  s  b  (a  b  c) / 2  b  (a  c  b) / 2  0
y  s  c  (a  b  c) / 2  c  (a  b  c) / 2  0
z  s  a  (a  b  c) / 2  a  (b  c  a) / 2  0
Y está claro que, por ejemplo, x  y  2s  b  c  a  b  c  b  c  a
 a  b  x  y  y  z  2 y  x  z  2z  c  c
y las otras dos condiciones se demuestran de forma similar.

10.3 Observación.
La Transformación de Ravi equivale geométricamente a determinar los puntos de
tangencia entre el triángulo y su circunferencia inscrita. (ver GA/11.4.10)

10.4 Observación.
Es posible expresar ciertos valores notables de la geometría del triángulo en función de
las x, y, z de la Transformación de Ravi:
abc
a) s   x yz
2
b) Por la fórmula de Heron: ABC   s(s  a)(s  b)(s  c)  ( x  y  z ) xyz

c) Puesto que ABC   s r  r 


ABC   ( x  y  z ) xyz

xyz
s x yz x yz
( x  y)( y  z )( z  x)
d) ABC  
abc
R
4R 4 x  y  z  xyz

Otras identidades que pueden ser útiles:


e) a  b  c  2( x  y  z)  (a  b  c)2  4( x  y  z)2
f) ab  bc  ca  x 2  y 2  z 2  3( xy  yz  xz )
En los siguientes problemas se pide demostrar la desigualdad indicada, y siempre se supone
que a, b, c son los lados de un triángulo.

10.5
 
2 a 2  b2  c 2  a  b  c 
2

10.6
ab  bc  ac  a 2  b2 c 2  2ab  bc  ac 

10.7
a b c bc a c  a b a bc
a)   3 b)   3
bca c a b a bc a b c

10.8
a bc  bc a  c  a b  a  b  c

y determinar cuándo ocurre la igualdad.

Asian Pacific Mathematics Olympiad 1996, Problema #5

10.9 M
Demuestra que, en todo triángulo ABC , se cumple R  2r , donde R es el circunradio
y r el inradio del triángulo.

10.10 D
a 2 (b  c  a)  b2 (c  a  b)  c2 (a  b  c)  3abc

IMO 1964 #2

10.11 M
ab  c  a   2bc

10.12 F
b  c  ac  a  ba  b  c  abc
10.13 F
a 2ba  b  b2cb  c   c 2ac  a   0

IMO 1983 #6
10.14 MF
3(ab  bc  ca)  (a  b  c)2  4(ab  bc  ca)
10.15 MF
ab  bc  ca  a 2  b2  c 2  2(ab  bc  ca)

10.16 D
3 a b c
   2
2 bc ca ab

10.17 D
     
a b2  c2  a 2  b c 2  a 2  b2  c a 2  b2  c2  3abc

10.18 M
a b bc ca 1
  
ab bc ca 8

10.19 MF
Si, además, se satisface ab  bc  ca  3 , demostrar que
3 abc  2 3

10.20 F
Si s es el semiperímetro del triángulo,
a) (s  a)(s  b)  ab
ab  bc  ca
b) ( s  a)(s  b)  ( s  b)(s  c)  ( s  c)(s  a) 
4

10.21 MD
Sea ABC un triángulo con AB  10 , BC  14 y CA  16 . Sea D un punto del interior
de BC . Denotamos por I B y I C los incentros de los triángulos ABD y ACD ,
respectivamente. Los circuncírculos de los triángulos BI B D y CI C D se cortan en los
puntos diferentes P y D. La máxima área posible de BPC se puede expresar de la
forma a  b c , donde a, b, c son enteros positivos y c no es divisible por el cuadrado
de ningún número primo. Determina a  b  c .

AIME I 2009 #15


11 Desigualdades geométricas con trigonometría.
11.1 M
Dado un triángulo ABC , demuestra que:
A B B C C A
a) tan tan  tan tan  tan tan  1
2 2 2 2 2 2
A B C 3
b) tan tan tan 
2 2 2 9

11.2 MF
Demuestra que en cualquier triángulo ABC
A a
sin 
2 2 bc

11.3 F
Dos circunferencias, 1 y 2 , tienen radio 5 y 12 respectivamente, y la distancia entre
sus centros es de 13 unidades. Las circunferencias se cortan en los puntos P y Q. Se
traza una recta l que pasa por P y corta la circunferencia 1 en X  P y 2 en Y  P .
Determina el máximo valor de PX  PY .

West Windsor Plainsboro Math Tournament 2013

11.4 F
Sea ABC un triángulo acutángulo. Demostrar que:

a) tan A  tan B  tan C  tan A tan B tan C .


b) tan A tan B tan C  3 3 .
11.5 M
Dado un triángulo ABC , demostrar que:
A a
a) sin 
2 bc
A B C 1
b) sin sin sin 
2 2 2 8
A B C A B C
c) sin 2  sin 2  sin 2  2 sin sin sin  1
2 2 2 2 2 2
A B C 3
d) sin 2  sin 2  sin 2 
2 2 2 4
A B C 9
e) cos 2  cos 2  cos 2 
2 2 2 4
A B C 3 3
f) cos cos cos 
2 2 2 4
A B C
g) csc  csc  csc  6
2 2 2

11.6
Supongamos que en un triángulo ABC se cumple

sin A  sin B  sin C  1

Demuestra que minA  B, B  C, C  A  30º

11.7 D
Sea ABCD un cuadrilátero convexo tal que BC  2 y CD  6 . Supongamos que los
baricentros de ABC , BCD y ACD forman los vértices de un triángulo equilátero.
Determina el máximo valor posible del área de ABCD.

(A) 27 (B) 16 3 (C) 12  10 3 (D) 9  12 3 (E) 30

AMC 12B 2019 #25


12 Desigualdades con las rectas del triángulo.
12.1 Comparación de ángulos y lados (GA/3.7.7)
Lado mayor determina ángulo contrario mayor y viceversa.

12.2 La desigualdad de Ptolomeo. (GA/13.2.7)


Dados cuatro puntos A, B, C, D se cumple
AB  CD  BC  DA  AC  BD
y la igualdad solo sucede si A, B, C, D están alineados o son cocíclicos en este orden.

12.3 Teorema y desigualdad de Euler. (GA/11.13.1)


Dado un triángulo ABC , sea O el circuncentro, I el incentro, r el inradio y R el
circunradio. Se cumple:
OI 2  R( R  2r ) , y por tanto R  2r

y la igualdad acontece si y solo si el triángulo es equilátero.

12.4 Teorema y desigualdad de Leibniz. (GA/11.13.4)


En un triángulo ABC de lados a, b, c , con circuncentro O, baricentro G y circunradio
R, se cumple:
OG2  R 2  a 2  b 2  c 2  , y por tanto 9R2  a 2  b2  c2
1
9

y la igualdad acontece si y solo si O  G , es decir, cuando el triángulo es equilátero.

12.5 F
Dado un triángulo ABC con la mediana AD , se cumple:
bca
a) AD 
2
1
b) Si AD  a , entonces BAC  90º .
2

12.6 F
Siendo a, b y c los lados de un triángulo y T su área, demostrar que
a 2  b2  c 2  4 3 T
¿Cuándo se da la igualdad?
IMO 1961 #2

12.7 D
Demostrar que en un triángulo ABC de lados a, b, c se cumple

4 3ABC  
9 abc
abc
12.8 D
Sea P un punto en el interior de un triángulo ABC con circunradio R. Sean p, q, r las
distancias de P a los lados BC, AC y AB, respectivamente. Demostrar que se cumple

a 2  b2  c 2
p q r
2R

12.9 F
Sea ABC un triángulo equilátero de lado a , sea P un punto del su interior y sean D, E
y F las proyecciones de M en los lados BC, CA y AB, respectivamente. Demostrar que:

a) PD  PE  PF  h , donde h es la altura del triángulo ABC . ("Lema de Viviani").


1 1 1 6 3
b)   
PD PE PF a
1 1 1 3 3
c)   
PD  PE PE  PF PF  PD a

12.10 F
Sean ha , hb y hc las longitudes de las correspondientes alturas por A, B y C de un
triángulo ABC dado, y sea r el radio de su incírculo. Demostrar que:
r r r
a)   1 b) ha  hb  hc  9r
ha hb hc

12.11 F
Sea ABC un triángulo con alturas AD, BE y CF, y sea H su ortocentro. Demostrar
que:
AD BE CF HD HE HF 3
a)   9 b)   
HD HE HF HA HB HC 2
12.12 MD
Las longitudes de los lados de un hexágono ABCDEF satisfacen AB  BC , CD  DE
y EF  FA . Demuestra que

BC DE FA 3
  
BE DA FC 2

IMO 1997 Shortlist #7

12.13 MD
Sea ABC un triángulo y sean L, M y N puntos en BC, CA y AB, respectivametne.
Sean P, Q y R los respectivos puntos de intersección de las rectas AL, BM y CN con el
circuncírculo de ABC . Demostrar que

AL BM CN
  9
LP MQ NR

COREA 1995

12.14 MF
Sean AD, BE y CF las alturas de un triángulo ABC , y sean PQ, PR y PS las distancias
entre un punto P a los lados BC, CA, AB, respectivamente. Demostrar que

AD BE CF
  9
PQ PR PS
12.15 M
Las cevianas AL, BM y CN de un triángulo ABC se cortan en un punto P. Demostrar
que
AP BP CP
  6
PL PM PN

si y solo si el punto P es el baricentro del triángulo.

12.16 MF
Las alturas AD, BE y CF cortan el circuncírculo de un triángulo ABC en los puntos
D', E' y F', respectivamente. Demostrar que:
AD BE CF AD BE CF 9
a)   9 b)   
DD' EE ' FF ' AD ' BE ' CF ' 4

12.17 D
Sean la , lb , lc las longitudes de las bisectrices internas de los ángulos de un triángulo, y
sean s y r el semiperímetro y el inradio del triángulo. Demostrar que:
a) la lb lc  r s 2
b) la2  lb2  lc2  s 2
c) la lb  lb lc  lc la  s 2

12.18 MF
Dado un triángulo ABC , sean M, N y P puntos arbitrarios en las rectas BC, CA y AB,
respectivamente. Si denotamos por a, b, c las longitudes de los lados del triángulo y por
R el circunradio, demostrar que
bc ca ab
   6R
AM BN CP

12.19 F
Demostrar que si R es el circunradio y denotamos por a, b, c las longitudes de los lados
del triángulo, se cumple:
1 1 1 1
   2
ab bc ac R
12.20 F
El lado AB del triángulo ABC tiene longitud 10. La bisectriz del ángulo A corta el
lado BC en D, y CD  3 . El conjunto de todos los posibles valores de AC es un
intervalo abierto m, n  . Determina m  n .

(A) 16 (B) 17 (C) 18 (D) 19 (E) 20

AMC 12B 2018 #12

12.21 M
Dadas dos circunferencias que se cortan en los puntos P y Q, demuestra como construir
un segmento AB que pasa por P cortando ambas circunferencias de forma que AP  PB
sea máximo.

USAMO 1975 #4
13 Inecuaciones.

13.1 M
Resuelve la siguiente inecuación:
4x2
 2x  9
1  1  2x 
2

IMO 1960 #2

13.2 D

Determina todos los números reales para los que se satisface la inecuación:
1
3  x  x 1 
2
IMO 1962 #2

13.3 F
Dados a  b  1 , resuelve la inecuación
ax  1 x  a

bx  1 x  b
14 Aplicación de las desigualdades en la resolución de ecuaciones.

El Apartado 22.7 del libro de Teoría de Números está dedicado a la aplicación de


las desigualdades en la resolución de ecuaciones diofánticas.

14.1 D
Determina todas las soluciones reales del siguiente sistema:
 x3  y 3  1
 4
 x  y 4  1
Soluciones.
1.2
a 2  b2 a b
Partimos de 1.1c : 2ab  a 2  b 2  2   
ab b a

1.3
1 sin 2 x  cos 2 x sin 2 x cos 2 x sin 2 x cos 2 x
     
sin x cos x sin x cos x sin x cos x sin x cos x sin x cos x sin x cos x
sin x cos x
  2
cos x sin x

1.4
Es un caso particular de 1.2 tomando b  1 .

1.5
x2  2 x2  1 1 1
   x2  1   2 por aplicación directa de 1.4
x 12
x 1
2
x 1
2
x2  1

1.6
Primer caso.
Supongamos que x, y son ambos positivos. Observamos que podemos suponer, sin pérdida de
generalidad, que y  x , es decir, estamos ante el caso 0  y  x  1 .

Entonces todos los valores absolutos son superfluos y la expresión se reduce a:


x y x y x  y x  y x  y 1  xy   x  y 1  xy 
 0   
1  xy 1  xy 1  xy 1  xy 1  xy 1  xy 
  2  2

x  x 2 y  y  xy 2  x  x 2 y  y  xy 2 2 y  2 x 2 y 2 y 1  x 2 
12  xy  1  xy  1  xy 
2 2 2

y esta última expresión es positiva porque todos sus factores son positivos.

Segundo caso.
Si los dos valores son negativos se puede reducir fácilmente al caso anterior:
x y x y (  x)  ( y ) x  y x y
   
1  xy 1  xy 1  ( x)( y) 1  ( x)( y) 1  x y

Tercer caso.
Finalmente, si uno es positivo y el otro negativo vemos que la expresión del enunciado
x y x y

1  xy 1  x y

Es una igualdad.
Fuente de esta solución: wpd.ugr.es/~jmmanzano/preparacion/problemas.php?c=4

1.7
En primer lugar vemos que

a  b 1  1   a  b a  b   a  b
2

a b  ab  ab
a b
1010  a, b  2020  1  , 2
1010 1010
a b
Haciendo el cambio de variable x  , y tenemos:
1010 1010
1010 x  1010 y 2  10102 x  y 2  x  y 2
1010 2 xy 1010 2 xy xy

Por lo tanto tenemos que demostrar que


x  y 2  9 para 1  x, y  2
xy 2
x  y 2  9  2x  y 2  9 xy  2 x 2
 2 y 2  4 xy  9 xy  2 x  2 y 2  5 xy
2

xy 2
x  y2 5
2 2
x y2 5 x y 5
       
xy 2 xy xy 2 y x 2
x
Con un nuevo cambio de variable z  , esta desigualdad se convierte en
y
1 5 1
z   para todo  z  2
z 2 2

Y vemos que, en efecto:


1 z2 1 5
z    2z 2  1  5 z  0  5 z  2z 2  1  2  z 2 z  1
z z 2

y es cierto porque los dos factores de esta última expresión son positivos.

Observación: Vemos que la igualdad se cumple en los casos z  2 y z  1/ 2 , que se


corresponden con los casos a, b  1010,2020 y a, b  2020,1010

1.8
Observamos que en la expresión ab  bc  cd aparecen b y c dos veces, y a y d solo una vez,
con lo que podemos especular que el valor máximo se obtendrá asignando b y c con valores
máximos.
Si no tuvieramos la limitación de trabajar con enteros, tendríamos valor máximo con
a  d  0, b  c  63 / 2  ab  bc  cd  992.25

Al trabajar con valores enteros y positivos tenemos máximo en


a  1 , b  30 , c  31 , d  1  ab  bc  cd  991
a  1 , b  31 , c  30 , d  1  ab  bc  cd  991

Pero todo esto son especulaciones. Ahora tenemos que demostrar que este es el valor máximo.
Para ello observamos la siguiente identidad:
ab  bc  cd  da  (b  d )(a  c)  ab  bc  cd  (b  d )(a  c)  da
Por otro lado, a  b  c  d  63  b  d  63  (a  c) , y por tanto
(b  d )(a  c)  da  (63  (a  c))(a  c)  da (*)

La expresión (63  (a  c))(a  c) se puede interpretar como el producto de dos números


positivos cuya suma es 63. Su valor máximo es 31 32 . También se puede encontrar el máximo
tomando x  a  c y viendo que la expresión f ( x)  (63  x) x representa una paràbola con
máximo en x  63 / 2 , luego los valores enteros máximos serán 31 y 32.
Por otro lado, d  a siempre será mayor o igual que 11 , así pues:

(*)  31 32  11  991

Ya hemos visto que esta igualdad se obtiene para a  1 , b  30 , c  31 , d  1 o para


a  1 , b  31 , c  30 , d  1 , con lo que el problema queda resuelto.

También se podría haber llegado a estos valores resolviendo los sistemas:


a  d  1
  a  d  1, c  31  b  63  1  31  1  30
a  c  32
a  d  1
  a  d  1, c  30  b  63  1  30  1  31
a  c  31

Llegando a los resultados anteriores.

Fuente de esta solución: “Cambridge Interview Question_ Trust your Intuition”

1.9
xy 12  x  y 2  xy 2  1  2xy  x 2  y 2  2xy  xy 2  1  x 2  y 2  1
y toma el valor mínimo 1 cuando x  y  0 , luego la respuesta correcta es D.

2.3
Aplicamos la desigualdad AM-GM tres veces:
ab 
 ab 
2

bc  (a  b)(b  c)(a  c)
 bc    ab bc ac  a 2b 2c 2  abc
2  8
ac 
 ac 
2 

2.4

1  a1 1  a2 ... 1  an   2n  1  a1 1  an2 ... 1  an   1


2
1  a1 1  a2 1  an
 ... 1
2 2 2

1  ai
Pero, aplicando la desigualdad AM-GM:  ai 1  ai
2
1  a1 1  a2 1  an
Luego ...  a1 a2 ... an  a1a2 ...an  1  1
2 2 2

2.5
a) Primera versión: Aplicando la desigualdad AM-GM:
a 2  b2 a2  c2 b2  c2
 a b  ab ,
2 2
 a c  ac ,
2 2
 b 2c 2  bc
2 2 2

Luego
a 2  b2 a 2  c 2 b2  c 2 2a 2  2b 2  2c 2
   ab  ac  bc   ab  ac  bc 
2 2 2 2
a 2  b 2  c 2  ab  ac  bc

Segunda versión:
a 2  b2  c 2  ab  bc  ac  a 2  b2  c 2  ab  bc  ac  0 
2a 2  2b2  2c 2  2ab  2bc  2ac  0  (a  b)2  (a  c)2  (b  c)2  0
Reduciendo la desigualdad al modelo 1.1b

b) Basta aplicar el apartado anterior teniendo en cuenta que


a  b  c2  a2  b2  c2  2(ab  bc  ac) , y por tanto:
ab  bc  ac  2(ab  bc  ac)  a 2  b2  c2  2(ab  bc  ac)  a 2  b2  c2  2(a 2  b2  c2 )

2.6
Utilizamos la siguiente factorización:
a3  b3  c3  3abc  (a  b  c)(a 2  b 2  c 2  ab  bc  ac) 

 a  b  c  a 2  b 2  c 2  ab  bc  ac  
Y aplicando el ejercicio anterior:
a 2  b 2  c 2  ab  bc  ac  a 2  b 2  c 2  ab  bc  ac   0 
a  b  c a 2  b2  c 2  ab  bc  ac   0  a3  b3  c3  3abc  0 
a 3  b3  c3  3abc

Ahora, mediante el siguiente cambio a  3 a , b  3 b , c  3 c llegamos a la desigualdad


deseada.

2.8
a  ...  an  a  ...  an 
n

Aplicamos la desigualdad AM-GM: n a1...an  1  a1...an   1 


n  n 

Aplicada a los números a1  1, a2  2 ,..., an  n , y teniendo en cuenta que


n(n  1)
1  2  ...  n 
2
 1  ...  n   n(n  1) / 2   n  1 
n n n

n! 1  2  ...  n       
 n   n   2 

2.9
Aplicamos la desigualdad AM-QM:
x yz x2  y 2  z 2

3 3
6 x yz x2  y 2  z 2 x2  y 2  z 2
2   42 
2
 12  4  3  x 2  y 2  z 2
3 3 3 3

2.10
(a  b  c) 2  a 2  b 2  c 2  2(ab  bc  ac)  1  2(ab  bc  ac) 
(a  b  c) 2  1
 ab  bc  ac
2

Aplicamos la desigualdad AM-QM:


abc a 2  b2  c 2
 a  b  c  3  a  b  c   3
1 3
  
2

3 3 3 3

(a  b  c) 2  1 3  1
Luego ab  bc  ac   1
2 2

 1 0  1 ( a  b  c) 2  1
Por otro lado: (a  b  c)2  0 , luego    ab  bc  ac
2 2 2

2.11
a 2  b2 a  b
  a 2  b2 
a  b2
ab 2 2
Aplicamos la desigualdad AM-QM:
ab

a 2  b2 ab

a 2  b2 a  b   a 2  b2
2 2

  
2 2  2  2 4 2


a  b
2
 a 2  b2
2

2.12
Sean x  b  c  a , y  a  c  b , y z  a  b  c .
Entonces
x y
x  y  b  c  a  a  c  b  2c  c  ,
2
xz yz
y de la misma manera: b  , a
2 2
Aplicamos la desigualdad AM-GM:
x y xz yz
xy   c , xz   b , yz  a
2 2 2
Luego
(b  c  a)(a  c  b)(a  b  c)  xyz 
x y xz yz
 x 2 y 2 z 2  xyxzyz  xy xz yz   abc
2 2 2

2.13
Aplicamos la desigualdad AM-GM:
ab a b ab
  2 
c c c c c
bc b c b c  ab bc ac ab bc ac
  2    8 
a a a a a c a b cc aa bb
ac a c ac
  2 
b b b b b 
ab bc ac abbcac
   8 8
c a b ccaabb

2.14
Aplicamos la desigualdad AM-GM por separado en cada paréntesis:
a 2b  b2c  c 2a  3 3 a 2bb2cc 2a  3 3 a3b3c3  3abcd
ab2  bc 2  ca 2  3 3 a3b3c3  3abcd

Multiplicando estas dos desigualdades llegamos al resultado deseado.

2.15
a a b
Nos vamos a basar en la siguiente desigualdad:    3a
b b c

En efecto, por la desigualdad AM-GM:


a a b aab
   33  3 3 a 2b 1c 1  3 3 a 2b 1c 1  3 3 a 2b 1c 1 3 1 
b b c bbc
 3 3 a 2b 1c 1 3 abc  33 a 3  3a
b b c c c a
y de la misma manera:    3b y    3c
c c a a a b

Por tanto
a b c a a b b b c c c a
3              3a  3b  3c  3(a  b  c) 
b c a b b c c c a a a b
a b c
   abc
b c a

2.16
A B
Aplicamos la desigualdad AM-GM dos veces: AB   2 AB  A  B
2
2 ab  xy  ab  xy 

  4abxy  2 ab  xy  2 ax  by  ab  xy ax  by 
2 ax  by  ax  by 

2.17
Aplicando la desigualdad AM-GM tres veces:

abc  ac  bc  


2 2 2  ac   bc  
2 2


2 
bca 2  ba  ca  
2 2 ba   ca   
2 2


2 
bac 2  bc  ac  
2 2 bc  2
 ac 2


2 

abc  bca  bac


2 2 2

ac   bc  ba   ca  bc   ac 
2 2

2 2

2 2

2 2 2

abca  b  c  
ac 2  bc 2  ba 2  ca 2  bc 2  ac 2 
2

abca  b  c  

2 ac   bc   ba 

2 2 2

2
abca  b  c   ac   bc   ba 
2 2 2

2.18
Aplicamos la desigualdad AM-GM:
4sin x  4cos x
2 2

 4sin x  4cos x  4sin x  cos x  41  2 


2 2 2 2

2
 4cos 4
2 2
4sin x x

Y la igualdad la encontramos cuando


2 2
sin 2 x  sin x   sin x 
4 sin 2 x
4 cos 2 x
 sin x  cos x 
2 2
1   1   1
cos 2 x  cos x   cos x 

 tan x   1  tan x  1  x  
2

2.19
Aplicando la desigualdad AM-GM,
a3 a3
 b  c  33 bc  3a
bc bc
b3 c3
Y de la misma manera,  a  c  3b y  a  b  3c .
ac ab

Sumando las tres igualdades siguientes tenemos


a3 b3 c3
bc ac  a  b  3a  3b  3c  3a  b  c  
bc ac ab
a 3 b3 c 3 a 3 b3 c 3
   2a  b  c   3a  b  c      abc
bc ac ab bc ac ab

2.20
Aplicando la desigualdad AM-GM:
a 3  a 3  b3  3 3 a 3a 3b3  3a 2b


  
b3  b3  c 3  3 3 b3b3c3  3b 2c   3 a 3  b3  c3  3 a 2b  b 2c  c 2 a 

c 3  c 3  a 3  3 3 c 3c3a 3  3c 2 a 

2.21
Aplicando la desigualdad AM-GM:
a3  ab2  2 a3ab2  2 a 4b2  2a 2b

Y de la misma manera:
b3  bc 2  2b2c
c3  ca 2  2c 2a

Y solo nos queda sumar las tres desigualdades anteriores.

2.22
Aplicamos la desigualdad AM-GM:
x3 y  2z y  2z x3 y  2z y  2z x3 x
  3   3 3 
 y  2z   y  2 z  27
3
2 2 2
27 27 27 27 3

Y por lo tanto:
x3 x y  2z y  2z 9x   y  2z    y  2z  9x  2 y  4z
    
 y  2 z  3 27
2
27 27 27

De la misma manera:
y3 9 y  2z  4x z3 9z  2x  4 y
 , 
z  2 x 2
27 x  2 y 2
27

Sumando las tres desigualdades anteriores llegamos a


x3 y3 z3 9x  2 y  4z 9 y  2z  4x 9z  2x  4 y
     
 y  2 z  z  2 x  x  2 y 
2 2 2
27 27 27
9 x  2 y  4 z  9 y  2 z  4 x  9 z  2 x  4 y 3x  3 y  3z x  y  z 3 1
    
27 27 9 9 3
Fuente de la solución: "2010 IMO Summer IMO Training: Inequalities Adrian Tang". pág. 2

2.23
Aplicando la desigualdad GM-AM,
bc

1
b  c   2 bc  2 bc
a a a a

Y aplicando el mismo principio a los otros dos sumandos llegamos a


bc ca ab  bc ac ab 
   2   
a b c  a ab c 
 bc ac   ac ab   ab bc 
   
   
    (*)
 a ab   ab c   c a 

De nuevo, aplicando la desigualdad GM-AM:


1/ 2
bc ac  bc ac 
  2  2 c
a ab  a ab 

Y aplicando el mismo principio a los otros dos sumandos llegamos a


 
(*)  2 a  b  c  a  b  c  a  b  c

Finalmente, basta tener en cuenta que, aplicando la desigualdad GM-AM:


a  b  c 3 a b c  
1/ 2

 3 abc 
1/ 2
3

2.24
9  a  b  c   a 2  b 2  c 2  2ab  bc  ac  
2


2ab  bc  ac   9  a 2  b 2  c 2 
 
a  b  c  ab  bc  ca  2 a  b  c  2ab  bc  ca   9  a 2  b 2  c 2   
 
2 a  b  c  a 2  b2  c 2  9

Lo cual es cierto, pues aplicando la desigualdad AM-GM:


a 2  2 a  a 2  a  a  3(a 2 a a )1 / 3  3a

Y de la misma forma con las otras dos variables llegamos a


 
2 a  b  c  a 2  b2  c2  3a  3b  3c  3(a  b  c)  3  3  9

2.25
Aplicando AM-GM:
1 1 1 1
1    33  abc  27
a b c abc
Luego
 1 1 1
ab  bc  ac  abc     abc  27
a b c
Aplicando nuevamente AM-GM:
a  b  c  3 3 abc  3 3 27  9

Luego, finalmente:
a  1b  1c  1  abc  ab  bc  ac  a  b  c  1  27  27  9  1  64
2.27
a b c abc
a bc 3     1 , y podemos aplicar 2.25:
3 3 3 3
1 a  b  c  1 32
2
a bbc c a  a b / 3bc / 3c a / 3  a  b  c  ab  bc  ac  
b c a 1
3
 1
3 3 3 3 3 3 3 3
a bb c c a  1

En donde hemos utilizado 2.5b.

2.28
Basta aplicar tres veces la desigualdad 2.25:
a abbc c  abbc c a  a cb a cb  aa  bb  cc  ab  bc  ca  ac  ba  bc 
 a 2  b2  c 2  2ab  bc  ac   a  b  c   12  1
2

2.29
Tomando x  a , y  b ,
a2 b2 x2 y 2 x3  y 3
  a b   x y  x y
b a y x xy
 x3  y 3  xy x  y   x 2 y  xy 2

Que se demuestra por Muirhead (2.30) puesto que


  
( 2 , 0 )   2 ,1   x3  y 3  x 2 y  xy 2
1
2
1
2

2.31
Observamos que
a  b  c3  3 3, 0 , 0   18 2 ,1, 0   361,1,1 
Luego queremos demostrar que
3 3 , 0 , 0   6 1,1,1   3 3 , 0 , 0   18 2 ,1, 0   361,1,1 
1
7

Es decir:
18
 3, 0 , 0    6  36  1,1,1   18  2 ,1, 0 
7  7  7

O equivalentemente:
18
 3 , 0 , 0    2 ,1, 0    6  36  1,1,1   0
7  7

Y esto se deduce de las inecuaciones  3, 0 , 0    2 ,1, 0  y 1,1,1   0

3.3
Basta aplicar la Desigualdad GM-HM:
a  x  a  b a 2  b2 x y x y
     
b  y  2 2 2 2

x y
 x y 2  2( x  y )
2

3.4
Utilizamos la desigualdad del problema 3.3. En nuestro caso
x  144  722  x  2( x  144  722  x)  2  578  34
Y la igualdad acontece cuando x 144  722  x  x  433

El dominio de definición de esta función es x  144 x  722  144,722

4.5
Tomamos a1  a2  ...  an  1 ,
1  1  ...  1b12  b22  ...  bn2   b1  b2  ...  bn 2 
n b12  b22  ...  bn2   b1  b2  ...  bn  
2

  b  b  ...  b   n b  bn  ...  b   b  b n ...  b


2 2 2
n b12  b22  ...  bn2  
2 1 2 n 1 2 n
1 2 n

n b 2
 b22 ...  b  b  b  ...  b
2
b  b  ...  b 2
b  b  ...  b
2 2
1 n
 1 2
 n

1 2 n 1 2 n
n2 n n n

4.6
1 2 3
Tomando a1, a2 , a3  x , y , z , y b1 , b2 , b3  , ,
x y z

y aplicando la desigualdad Cauchy-Schwarz, tenemos:


1 4 9
   x  y  z      1  2  3  36
1 4 9 2

x y z x y z

1 1 1
  , o x, y, z    , , 
x y z
Y la igualdad solo ocurre cuando
1 2 3 6 3 2
4.7
  
f ( x)2  a sin x  b cos x   a 2  b2 sin 2 x  cos 2 x  a 2  b2  f ( x)  a 2  b2
2

 tan x  x  arctan a / b
a sin x
y este máximo aparece para 
b cos x

4.8
Por la Desigualdad Cauchy-Schwarz:

a  b  c  d 2  12  12  12  12 a 2  b2  c 2  d 2  
8  e2  416  e2  
64  16 e  e2  64  4e2  16 e  5e 2  0  e(16  5e)  0  0  e  16 / 5
0
e(16  5e)  0  e  
16 / 5

16
El valor máximo es e  , que ocurre cuando (a, b, c, d )  k (1,1,11)  a  b  c  d , y por
5
16 6
tanto 4a   8  a  b  c  d 
5 5

4.9
32  1  a  1  b  1  c   a 2  b 2  c 2 1  1  1  3a 2  b 2  c 2  
2

3  a 2  b2  c 2

4.10

Tomando u  a b , b c , c a  y v c b ,a c ,b a 
a b    b c    c a   a b  b c  c a
2 2 2 2 2 2

c b   a c    b a   c b  a c  b a
2 2 2 2 2 2

 a bc b  b ca c  c ab a   abc  abc  abc 2 2


 3abc   9a 2b2c 2
2

Y es, por tanto, una aplicación directa de la desigualdad Cauchy-Schwarz.

Observación. Este problema se podría haber resuelto aplicando la desigualdad GM-HM:

Dividiendo los dos lados por a 2b2c 2 , la desigualdad se transforma en


 a b c  b c a 
        9
 c a b  c a b 

a b c
 
a b c a b c
Y aplicando la Desigualdad GM-HM: 1  3 1  3  c a b 3  
cab 3 c a b
b c a
Y de la misma forma 3   
c a b
4.11
1 1 4 16 bcd  acd  4abd  16abc 64
     
a b c d abcd abcd
a  b  c  d bcd  acd  4abd  16abc   64abcd

Tomando las cuaternas u  ( a , b , c , d ) y v  ( bcd , acd , 4abd , 16abc )


u  v2     8   64abcd
2 2
abcd  abcd  2 abcd  4 abcd abcd

y vemos que es una aplicación directa de la desigualdad Cauchy-Schwarz.

4.12
Aplicamos la Desigualdad Cauchy-Schwarz "en la forma de Engel" 4.3:
 a2 c2  2
 2  2
b2
 2 
 a  1  b 2  1  c 2  1  a  b  c 2  
 a 1 b 1 c 1
a 2  b 2  c 2  3  a  b  c  
2

3  a  b  c   a 2  b 2  c 2  2ab  bc  ca 
2

4.13
Basta elevar al cuadrado y desarrollar algebraicamente:
a12  b12  a22  b22  a1  a2 2  b1  b2 2 

a 2
1  b12  a22  b22   a  a   b  b  
2
1 2
2
1 2
2

  
a12  b12  a22  b22  2 a12  b12 a22  b22  a1  a2   b1  b2  
2 2

a12  b12  a22  b22  2 a 2


1  b a 1
2 2
2 b  a
2
2
2
1  a22  2a1a2  b12  b22  2b1b2 
  
2 a12  b12 a22  b22  2a1a2  b1b2  
a2
1  
 b12 a22  b22  a1a2  b1b2 
2

Que es la desigualdad Cauchy-Schwarz.

4.14
Observamos que
( a  b) 2 a 2  b 2  2ab a 2  b2  2c 2  2ab  2c 2 2ab  c 2 
  1 2
a 2  b 2  2c 2 a 2  b 2  2c 2 a 2  b 2  2c 2 a  b2  2c 2

Luego
( a  b) 2 (b  c) 2 (c  a ) 2
  
a 2  b 2  2c 2 b 2  c 2  2a 2 c 2  a 2  2b 2
 ab  c 2 bc  a 2 ca  b 2 
 3  2 2   
 a  b  2c
2 2
b 2  c 2  2a 2 c 2  a 2  2b 2 
Luego nuestro problema se reduce a demostrar que
(a  b) 2 (b  c) 2 (c  a ) 2
  3
a 2  b 2  2c 2 b 2  c 2  2a 2 c 2  a 2  2b 2

Aplicamos la desigualdad Cauchy-Schwarz "en la forma de Engel" (4.3):


( a  b) 2 a2 b2
 
a 2  b 2  2c 2 a 2  c 2 b 2  c 2

Y haciendo lo mismo en los otros dos sumandos llegamos a:


( a  b) 2 (b  c) 2 (c  a ) 2
  
a 2  b 2  2c 2 b 2  c 2  2a 2 c 2  a 2  2b 2
a2 b2 b2 c2 c2 a2 a 2  c2 b2  c2 b2  a 2
        3
a 2  c2 b2  c2 b2  a 2 c 2  a 2 c2  b2 a 2  b2 c 2  a 2 b2  c2 b2  a 2

Fuente de la solución: "Secrets in Inequalities (volume 1)" (Pham Kim Hung) pág. 35

4.15
1 1 1 1 1 1 x 1 y 1 z 1
1  2       1   1   1     
x y z x y z x y z
 x 1 y 1 z 1
x  y  z     x  y  z  
 x y z 

  
 x 1 2  

y  1
2
 

z  1 
2

 
x  y  z  x 1  y 1  z 1
2

 x y z 
 
En donde hemos aplicado la desigualdad Cauchy-Schwarz "en la forma de Engel" (4.3).

4.16
x  2 y  2  3z  6  x  2( y  1)  3( z  2) 
 x  2 y 1  3 z  2

Aplicamos la desigualdad Cauchy-Schwarz:


 
x  2 y  1  3 z  2  1  2  3x  y  1  z  2  6  6  62 
2

x  2 y 1  3 z  2  6

Y la igualdad acontece cuando las ternas son proporcionales:


x y 1 z  2 2 x  y  1
  
1 2 3 3x  z  2

y  1
x  y  z  3  x  2 x  1  3x  2  3  6 x  3  3  x  1  
z  1

Es decir, cuando x  y  z  1 .

Observación: En las soluciones oficiales (SE pág. 1042) se presenta un razonamiento


alternativo mediante la desigualdad de Jensen aplicada a la función f ( x)  x .
5.1
a b a b  0
  (a  b)( x  y )  0
x  y  x  y  0

0  (a  b)( x  y)  ax  ay  bx  by  ax  by  bx  ay   ax  by  bx  ay

5.2
Puesto que la desigualdad es simétrica en las dos variables, podemos suponer x  y .
1 1 1 1
x y 0   
y x y x
x2 y2 1 1 1 1 1 1
  x2  y 2  x2  y2 x y
y x y x y x y x

Y, aplicando el principio de Reordenación,


1 1 1 1 1 1 x2 y2
x y x y  x2  y2    x y
y x x y x y x y

5.3
Desarrollando la expresión algébrica tenemos
0  (a  b)(c  d )  (a  c)(b  d )  (d  a)(b  c) 
 ac  ad  bc  bd  ab  ad  bc  cd  bd  cd  ab  ac 
 2ac  ad  bc  bd 

Primera versión.
0  2ac  ad  bc  bd   0  ac  ad  bc  bd  ac  bd   ad  bc  
ac  bd  ad  bc
Si a  b y c  d , esta última desigualdad es una aplicación directa del Principio de
Reordenación.
Si a  b y c  d entonces a  d  a  c  b  c  a  d  b  c , contradiciendo la hipótesis del
enunciado a  d  b  c .
De la misma manera, si a  b y c  d , entonces
a  b  a  d  b  d  b  c  a  d  b  c , contradiciendo de nuevo la hipótesis
ad bc.
Finalmente, el caso a  b y c  d implica, por el principio de la Reordenación,
bd  ac  bc  ab , y por lo tanto se satisface la desigualdad propuesta.

Segunda versión.
Basta tener en cuenta que ac  ad  bc  bd  (a  b)(c  d )
y que a  d  b  c  a  b  c  d
Luego ac  ad  bc  bd  (a  b)2  0

5.4
Primera versión.
Sean xi  0 y c  n x1  ...  xn
x1 xx xx x x ... x
Sean a1  , a2  1 2 2 , a3  1 23 3 , ... , an  1 n n  1
c c c c
1 1 1
y b1  , b2  , ... , bn  1
a1 a2 an

Las secuencias ai  y bi  están ordenadas de forma opuesta. Luego:


a1b1  ...  anbn  a1bn  a2b1  a3b2  ...  anbn 1
x1 x1 x2 c x1 x2 x3 c 2 x x x x x  x  x3  ...  xn
1  ...  1   2  3
 ...  1  2  3  ...  n  1 2
c c x1 c x1 x2 c c c c c
x  x  x3  ...  xn
c n  x1  x2  x3  ...  xn  n x1  ...  xn  1 2
n

Segunda versión.
Sin pérdida de generalidad podemos suponer que a1a2 ...an  1 (ver Normalización, Tema 7).
x x x
Realizamos el cambio de variable a1  1 , a2  2 , ..., an 1  n 1 , con x1, x2 , ..., xn  0 ,
x2 x3 xn
x x x x x
Luego an  n y el problema se reduce a demostrar 1  2  ...  n 1  n  n
x1 x2 x3 xn x1

Observamos que la secuencia ( x1, x2 ,..., xn ) es creciente, mientras que la secuencia


1 1 1
 , ,...,  es decreciente. Por lo tanto, aplicando el Principio de Reordenación,
 x1 x2 xn 
x1 x2 x x 1 1 1 1 1 1 1
  ...  n 1  n  x1  x2  ...  xn 1  xn  x1  x2  ...  xn  n
x2 x3 xn x1 x2 x3 xn x1 x1 x2 xn

5.5
Las secuencias (a, b, c) y (a, b, c) están ordenadas de la misma manera. Luego
a b c  a b c 
a 2  b2  c 2      ab  bc  ac
 a b c  b c a 

y la igualdad se produce si y solo si a  b  c

5.6
a b c  a b c
a 3  b3  c 3   2   ac 2  ba 2  cb 2
a b 2
c 2  c 2 a 2
b 2 

5.7
Sean a, b, c números reales positivos.
 1 1 
Las secuencias a, b, c  y 
1
, ,  están ordenadas de la misma manera. Luego
bc ca a b
 a b c   a b c 
 1 1 1  1 1 1 
 b  c ca a  b   c  a ab b  c 

 a b c   a b c 
 1 1 1  1 1 1 
 b  c ca a  b   a  b bc c  a 

Y sumando estas dos desigualdades llegamos a

 a b c   a b c   a b c 
2 1 1 1  1 1 1  1 1 1 
 b  c c  a a  b   c  a a  b b  c   a  b b  c c  a 
 a b c  a b c a b c
2         
bc ca a b c a a b bc a b bc ca
ab bc ac
   
ab bc ca
a  b b  c c  a   (b  c)(a  b)(c  a)  (a  c)(b  c)(c  a)  3 
a  b b  c c  a 
a b c 3
  
bc ca ab 2

que es la desigualdad de Nesbitt (3.6).

5.8
 1 
Las secuencias ai  y   están ordenadas de la misma manera:
 s  ai 
1 1
an  am  s  an  s  am  
s  an s  am

Luego:

 a1 ... an   a1 a2 ... an 
 1 1  1 1 1  k  2, 3, ..., n
 ...   ... 
 s  a1 s  an   s  ak s  ak 1 s  ak 1 

Luego, sumando las n  1 desigualdades anteriores, llegamos (!!!!) al resultado deseado.

5.9
Primera versión.
(a  b)(b  c)  ab  ac  b 2  bc  b(a  c)  ac  b 2 
ac  b 2 (a  b)(b  c)  b(a  c) (a  b)(b  c)
  b
ac ac ac

Y de la misma forma
bc  a 2 (a  b)(a  c) ab  c 2 (b  c)(a  c)
 a y  c
bc bc ab ab

Luego
a 2  bc b 2  ac c 2  ab
  
bc ac ab
(a  b)(b  c) (a  b)(a  c) (b  c)(a  c)
b a c  abc 
ac bc ab
(a  b)(b  c) (a  b)(a  c) (b  c)(a  c)
   2(a  b  c)
ac bc ab

Podemos suponer, sin pérdida de generalidad, que a  b  c .


Y por tanto:
1 1 1
ab ac bc  
bc ac ab

Y también (a  b)(a  c)  (a  b)(b  c)  (b  c)(a  c) .


Luego, aplicando el criterio de Reordenación,
1 1 1
(a  b)(a  c)  (a  b)(b  c)  (b  c)(a  c) 
bc ac ab
1 1 1
(a  b)(a  c)  (a  b)(b  c)  (b  c)(a  c)  2(a  b  c)
ac ab bc

tal y como queríamos ver.

Segunda versión.
Podemos suponer, sin pérdida de generalidad, que a  b  c .
1 1 1
Luego: a 2  b2  c 2 , y también a  b  a  c  b  c    .
bc ac ab

Luego, aplicando el criterio de Reordenación,


a2 b2 c2 a2 b2 c2
    
bc ca ab ab bc ca

Y por tanto:
a 2  bc b 2  ac c 2  ab
  
bc ac ab
a2 b2 c2 bc ac ab
     
bc ac ab bc ac ab
a2 b2 c2 bc ac ab
     
ab bc ca bc ac ab
a 2  ab b 2  bc c 2  ac
   abc
ab bc ca

Fuente de esta versión: Secrets in Inequalities (volume 1) (Pham Kim Hung), pág. 92.
5.11
Primera versión.
Supongamos la secuencia ordenada a  b  c  d .
Entonces a 2  b2  c 2  d 2 ,
Y también b  c  d  c  d  a  d  a  b  a  b  c ,
a2 b2 c2 d2
Y finalmente se cumple   
bcd cd a d ab abc

Entonces, aplicando la versión "inversa" de la desigualdad de Chebyshev:


 a2 b2 c2 d2 
4  a 2  b 2  c 2  d 2  exp 
1
   
4 bcd cd a d ab a bc

Con exp  b  c  d   c  d  a   d  a  b  a  b  c  3(a  b  c  d )

Así pues,
 2 2
c2 d2 
4
3
a  b  c  d  a  b   
4 bc d cd a d a b a bc
 a2 b2 c2 d2 
 a  b  c  d 
16
    
3  b  c  d c  d  a d  a  b a  b  c 

Ya solo queda demostrar que a  b  c  d  4 , lo cual es cierto por el Corolario a la


desigualdad Cauchy-Schwarz:

4  a 2  b2  c 2  d 2 
a  b  c  d 2  16  42  a  b  c  d 2  4  a  b  c  d
4

Segunda versión.
Supongamos la secuencia ordenada a  b  c  d .
Entonces a 2  b2  c 2  d 2 ,
Y también b  c  d  c  d  a  d  a  b  a  b  c
1 1 1 1
Luego   
bcd cd a d ab abc

Aplicamos la versión "directa" de la desigualdad de Chebyshev:


a2 b2 c2 d2
 exp 1 exp 2 
1
  
bcd cd a d ab abc 4

Donde exp 1  a 2  b2  c 2  d 2  4
1 1 1 1
Y exp 2    
bcd cd a d ab abc

Ahora, se demuestra (!!!!) que


exp 2 
 
4 a 2  b2  c 2  d 2

16

16 4

3a  b  c  d  3a  b  c  d  3  4 3
Fuente de la segunda versión: Secrets in Inequalities (volume 1) (Pham Kim Hung), pág. 54

6.5
Sea f ( x)  x . Esta función es cóncava, y por tanto podemos aplicar la Desigualdad de
Jensen:
ab  bc  ac  f ab  bc  ac   a f (b)  b f (c)  c f (a)  a b  b c  c a

Y ahora aplicamos
3ab  bc  ac   (a  b  c) 2  12  1  ab  bc  ac 
1
3

Y aplicamos ahora que la función f ( x)  x es estrictamente creciente:


1 1 1
ab  bc  ac   ab  bc  ac  
3 3 3

1
Así pues,  ab  bc  ac  a b  b c  c a
3

6.6
Sean a1, a2 , ..., an números reales positivos.
Si uno de los valores ai es cero, la desigualdad AM-GM se cumple trivialmente. Luego
podemos suponer que a1, a2 , ..., an  0 .
Consideremos la función f ( x)  log( x) . Es una función estrictamente creciente. Luego
a1  a2  ...  an n
 a1 a2 ... an 
n
log a1   log a2   ...  log an 
 a  a2  ...  an 
log 1  
  log n a1 a2 ... an  log a1 a2 ... an  
1
 n  n n
Es decir, queremos demostrar que
 a  a2  ...  an  f (a1 )  f (a2 )  ...  f (an )
f 1 
 n  n

Que es precisamente la desigualdad de Jensen aplicada a la función cóncava f ( x)  log( x) .

6.7
10
 1
Observamos que la función f ( x)   x   es convexa en 0, , pues su segunda derivada
 x
es positiva. Aplicamos la desigualdad de Jensen:

10 10 10
 1  1  1  f (a)  f (b)  f (c) 
 a     b     c    f (a)  f (b)  f (c)  3 
 a  b  c  3 
  a  b  c    1  
10 10
1  10  1010
3 f     3 f     3 3    3   9
  3    3   3 3 3
1
Este mínimo se alcanza cuando a  b  c  . En efecto:
3
10 10 10 10 10
 1  1  1 1   10  1010
 a     b     c    3  3   3   9
 a  b  c 3  3 3

6.8
Sabemos que en todo triángulo ABC se cumple A  B  C   , luego
1 1 1 
A  B  C 
3 3 3 3
1 1 1
Luego t1  t2  t3     1 y x1 , x2 , x3  A, B, C cumplen las condiciones de la
3 3 3
desigualdad de Jensen.

Por otro lado, f ( x)  sin x es una función cóncava en 0  x  180º , luego:

  1  1
 sin   sin A  B  C   sin A  sin B  sin C  sin A  sin B  sin C 
3 1 1 1 1 1
2 3 3 3 3  3 3 3 3

O equivalentemente:
3 3
sin A  sin B  sin C 
2

6.9
Siguiendo con los elementos introducidos anteriormente,
f ( x)  tan x es una función convexa en 0  x  90º , luego:

  1  1
3  tan   tan A  B  C   tan A  tan B  tan C  tan A  tan B  tan C 
1 1 1 1 1
3 3 3 3  3 3 3 3

O equivalentemente:
3 3  tan A  tan B  tan C

6.10
A B C 
0  A, B, C    0  , ,  , y por tanto
2 2 2 2
1  A B C  A B C A B C
3 3  3 tan(30)  3 tan   3 tan     tan  tan  tan
3  6  6 6 6 2 2 2

Es decir:
A B C
3  tan  tan  tan
2 2 2

Con la igualdad si y solo si A  B  C  60º .

6.11
a1a2 ...an (1  a1 )(1  a2 )...(1  an )
 
a1  a2  ...  an  n  a1  a2  ...  an n
n

a1  a2  ...  an n   a1  a2  ...  an  (*)


n
a1a2 ...an

(1  a1 )(1  a2 )...(1  an ) n  a1  a2  ...  an n  n  a1  a2  ...  an 

La función f ( x)  ln( x) es creciente, luego

n
 a1a2 ...an   a  a  ...  an 
(*)  ln    ln  1 2  
 (1  a1 )(1  a2 )...(1  an )   n  a1  a2  ...  an 
n
 ai   a1  a2  ...  an   (a1  a2  ...  an ) / n 
 ln 1  a   n ln n  a   n ln   
i 1  i   1  a2  ...  an   (n  a1  a2  ...  an ) / n 
n
  a  a  ...  an   a  a  ...  an 
 ln a i  ln 1  ai   n ln  1 2   ln 1  1 2 
i 1   n   n 

Tomando la función f ( x)  ln x   ln 1  x  , la desigualdad anterior equivale a demostrar


n
 a  a  ...  an 
i 1
f (ai )  n f  1 2
 n

Y, aplicando la desigualdad de Jersen, se reduce a demostrar que f (x) es convexa en


 1 / 2 , 1 :

f ( x)  ln x   ln 1  x   f ' ( x)  x 1  (1  x) 1 (1)  x 1  (1  x) 1 
1 1
 f ' ' ( x)  (1) x 2  (1)(1  x) 2 (1)   0
x 2
(1  x) 2
1 1 1
 2  x 2  (1  x) 2  1  2 x  x 2  0  1  2 x  2 x  1  x 
(1  x) 2
x 2

Fuente de esta solución: Secrets in Inequalities (volume 1) (Pham Kim Hung) pág. 70.

6.17
x1 x x
A( x1 , x2 ,..., xn )   2 2  ...  2 n  x1 f ( x2 )  x2 f ( x3 )  ...  xn f ( x1 ) (*)
x  1 x3  1
2
2 x1  1

, x  0,2
1
donde f ( x)  2
x 1
x
Aplicando el “truco de la recta tangente”, f ( x)  g ( x)  1 
2
y por tanto
 x   x   x 
(*)  x1 g ( x2 )  x2 g ( x3 )  ...  xn g ( x1 )  x1 1  2   x2 1  3   ...  xn 1  1  
 2  2  2
x x  x x  ...  xn x1 1 3
 x1  x2  ...  xn  1 2 2 3  2 
2 2 2

En donde hemos utilizado la siguiente propiedad:

x1x2  x2 x3  ...  xn x1 
 x1  x2  ...  xn  22
2
 1
4 4

Este mínimo se alcanza cuando xk  xk 1  1 y todos los demás x j  0 .

Fuente de esta solución: https://dgrozev.wordpress.com/2020/04/16/а-weaker-jensens-inequality-an-olympiad-approach/

7.7
Vemos que la desigualdad es homogénea de grado 2 en todos sus términos excepto en la
constante 1. Pero aprovechando la condición a  b  c  1 , tenemos 1  12  a  b  c  , y la
2

podemos transformar en la siguiente desigualdad equivalente:


a 2  b 2  c 2  a  b  c   4ab  bc  ca  
2

a 2  b 2  c 2  a 2  b 2  c 2  2ab  bc  ca   4ab  bc  ca  
 
2 a 2  b 2  c 2  2ab  bc  ca  
a 2  b 2  c 2  ab  bc  ca
Que es una desigualdad ya demostrada.

7.8
abc
En primer lugar observamos que abc  1  3 abc  1  a  b  c  3
.
abc
x y z
, b , c
Ahora realizamos la siguiente sustitución: a 
y z x
a b c xz xy yz x y z
   abc  2  2  2   
b c a y z x y z x

Esta última desigualdad se demuestra fácilmente mediante el método de reordenación:


xz xy yz 1 1 1 1 1 1 z x y
2
 2  2  xz 2  xy 2  yz 2  xz 2  xy 2  yz 2   
y z x y z x x y z x y z
7.9
Vemos que esta inecuación no es homogénea, y procedemos a homogeneizarla:
abc  1  abc   11 / 3  1
1/ 3

a  b  c  a 2  b2  c2  a  b  c abc   a 2  b2  c 2
1/ 3

Ahora desarrollamos la parte de la izquierda:


a  b  cabc1/ 3  a4 / 3b1/ 3c1/ 3  a1/ 3b4 / 3c1/ 3  a1/ 3b1/ 3c4 / 3
Luego queremos demostrar:
a4 / 3b1 / 3c1 / 3  a1 / 3b4 / 3c1 / 3  a1 / 3b1 / 3c4 / 3  a2  b2  c2

Sabemos que es cierta por Muirhead puesto que  4 / 3,1/ 3,1/ 3    2 , 0 , 0  .

Aplicamos la desigualdad AM-GM apropiada:


2a 2 b 2 c 2 3a 2 b 2 c 2 4a 2  b 2  c 2
      
3 6 6 6 6 6 6
a 2  a 2  a 2  a 2  b2  c2 6 2 2 2 2 2 2 6 8 2 2
  a a a a b c  a b c  a 4 / 3b1 / 3c1 / 3 
6
2a 2 b 2 c 2
a 4 / 3b1 / 3c1 / 3   
3 6 6
De la misma manera:
a 2 2b2 c 2 a 2 b2 2c 2
a1 / 3b 4 / 3c1 / 3    , a1 / 3b1 / 3c 4 / 3   
6 3 6 6 6 3

Y ya solo falta sumar las tres desigualdades anteriores, puesto que:


2a 2 b 2 c 2 a 2 2b 2 c 2 a 2 b 2 2c 2
        
3 6 6 6 3 6 6 6 3
4a 2 b 2 c 2 a 2 4b 2 c 2 a 2 b 2 4c 2
        
6 6 6 6 6 6 6 6 6
6a 2  6b 2  6c 2
  a 2  b2  c2
6

7.11
Puesto que la desigualdad es homogénea, podemos suponer que abc  1 , y por tanto:
1 1 1
 3 3  3 1
a  b  1 b  c  1 c  a3  1
3 3

Con el cambio de variable x  a3 , y  b3 , z  c3 obtenemos la desigualdad equivalente


1 1 1
  1
x  y 1 y  z 1 z  x 1

Y finalmente, con el cambio de variable A  x  y  1 , B  y  z  1, C  z  x  1 llegamos


finalmente a
1 1 1
  1
A B C

1 1 1 BC  AC  AB
  1  1  BC  AC  AB  ABC 
A B C ABC
 ABC  BC  AC  AB  0  ( A  1)( B  1)(C  1)  ( A  B  C )  1  0 
 ( x  y )( y  z )( z  x)  (2 x  2 y  2 z  3)  1  0
 ( x  y )( y  z )( z  x)  2( x  y  z )  2  0
 ( x  y )( y  z )( z  x)  2( x  y  z )  2 (*)
Ahora utilizamos la siguiente identidad:
( x  y)( y  z)( z  x)  2( x  y  z)  xyz  ( x  y  z)( xy  yz  zx  2)

Luego
(*)  ( x  y  z)( xy  yz  zx  2)  3 
x yz 3
Finalmente, aplicamos la desigualdad AM-GM:  xyz  1
3
xy  yz  zx 3 2 2 2
 x y z  ( xyz )2 / 3  1  xy  yz  zx  3  xy  yz  zx  2  1
3
y ya solo nos queda multiplicar las dos desigualdades anteriores.

7.12
Vemos que
a b c
f (a, b, c)    1
a 2  8bc b2  8ca c 2  8ab

es una función homogénea de grado 0, luego podemos añadir la restricción a  b  c  1 .

En efecto, supongamos que a  b  c  m  0  1 . Entonces

abc a b c
1   
m m m m

Y realizamos el cambio de variable a' a / m , b' b / m , c' c / m , obteniendo una


desigualdad equivalente a la primera:

a b c
f a' , b' , c'  f  , ,   f a, b, c 
m m m

Nota: La solución de este problema sigue en 6.4.

7.14
x y z
Realizaremos la misma sustitución que en el problema 7.2: a  , b , c
y z x
x z  y x  z y
  1    1    1    1 
y y  z z  x x
 x  y  z  y  z  x  z  x  y 
    1
 y  z  x 
x  y  z  y  z  x z  x  y   1 
x  y  z  y  z  x z  x  y   xyz
Y esta desigualdad es el problema 2.12.

8.1
1 2 3 4 5 6
a) Observamos que  ,  ,  ... luego elevando al cuadrado, tenemos
2 3 4 5 6 7
2
 1 3 5 7 99  1 1 3 3 5 5 7 7 99 1 2 3 4 5 6 7 8 99
    ...          ...         ... 
 2 4 6 8 100  2 2 4 4 6 6 8 8 100 2 3 4 5 6 7 8 9 100
1 2 3 4 5 6 7 8 99 1 1 3 5 7 99 1 1
        ...      ...  
2 3 4 5 6 7 8 9 100 100 2 4 6 8 100 100 10

b) Por inducción en n:
1 1 1 1
Si n  1 :    cierto.
2 3 1  1 4 2
1 3 1 1 3 1
Si n  2 ;       3 7  8  9  7  64 cierto.
2 4 3 2 1 7 8 7

Suponiendo que es cierto para n, es decir:


1 3 5 7 2n  1 1
   ... 
2 4 6 8 2n 3n  1

queremos demostrar que también lo es para n+1:


 1 3 5 7 2n  1  2n  1 1
    ...   
2 4 6 8 2n  2n  2 3(n  1)  1
1 2n  1 1
  (2n  1) 3(n  1)  1  3n  1(2n  2) 
3n  1 2n  2 3(n  1)  1
 
(2n  1) 2 3(n  1)  1  3n  1(2n  2) 2  4n 2  4n  1 3n  4  3n  1(4n 2  8n  4) 
12n  28n  19n  4  12n  28n  20n  4  0  n
3 2 3 2

Lo cual es obviamente cierto.


1 1
c) Aplicando el apartado anterior: 122  144  151  12  144  151  
155 12

8.2
Las secuencias a, b, c, d  y (a 2 , b2 , c 2 , d 2 ) están ordenadas de la misma manera, luego
podemos aplicar la desigualdad de :
 a  b  c  d   a  b  c  d  a a  bb  c c  d d a 3  b3  c 3  d 3
2 2 2 2 2 2 2 2
      
 4   4  4 4

Podemos volver a aplicar la d. s con la secuencias a, b, c, d  y a, b, c, d  :

a  b  c  d 2  a  b  c  d   a  b  c  d  a a  bb  c c  d d a  b  c  d
   
2 2 2 2
  
16  4   4  4 4

Uniendo estas dos desigualdades llegamos a:

 a  b  c  d  a  b  c  d   a  b  c  d   a  b  c  d  a  b  c  d
2 2 2 2 2 3 3 3 3
       
 4  16  4   4  4

Es decir:
a  b  c  d 3  a3  b3  c3  d 3  a  b  c  d 3  a3  b3  c3  d 3
4  16 4 16

Tal y como queríamos ver.

8.3
Aplicamos la desigualdad Cauchy-Schwarz:
2

 
2 1
9
1 1
9a  1  9b  1  9c  1   3 a   3 b   3 c   
9 9

 2
1 
2
1  1  
2

 

2 2

 3  3  3  a     b     c   
2

9  9  9 
 
 1 1 1  3
 3  32  a   b   c    3  32  a  b  c   
 9 9 9  9
 3
 3  32 1    36  9a  1  9b  1  9c  1  36  6
 9

Nota: Un desarrollo más limpio sería comparar las ternas  


9a  1, 9b  1, 9c  1 y (1,1,1) y
aplicar la desigualdad Cauchy-Schwarz:

9a  1  9b  1  9c  1  1  1  1 9a  1  9b  1  9c  1  3 9(a  b  c)  3  6

8.4
Aplicando la desigualdad Cauchy-Schwarz:
abc  1
2 2 2
1 1  1 1 1  1 1 1 1 1 1
      
  2      
     
 abc   bc ac ab   c a 2 c 2   b2 c 2 a 2   b2 c 2 a 2 
abc 1 1 1
 2 2 2
abc b c a
8.5
2 2 2 2 2 2
 a   b   c  b 2 c 2  a  a 2 c 2  b  a 2b 2  c 
 
  
            
 1  ab   1  bc   1  ac  b 2c 2  1  ab  a 2c 2  1  bc  a 2b 2  1  ac 
2 2 2 2 2 2
 abc   abc   abc   1   1   1 
            
 bc  abbc   ac  acbc   ab  abac   bc  b   ac  c   ab  a 
1 1 1
 2  2  2
b 1  c  c 1  a  a 1  b 
2 2 2

Aplicamos la desigualdad AM-GM:


1 1 1 1 1 1
   3 
b 2 1  c  c 2 1  a  a 2 1  b  b 2 1  c  c 2 1  a  a 2 1  b 
3
2 2 2 2 2 2

1 1
33 33
a b c 1  a  1  b  1  c  1  a  1  b 2 1  c 2
2 2 2 2 2 2 2

 4  3 1  a  1  b  1  c 
1 3
3 
2 2 2

3
1  a 2 1  b 2 1  c 2 4

1 1 1 1 1 1
 2  2 33 2 
b 1  c  c 1  a  a 1  b  b 1  c  c 1  a  a 1  b 
2 2 2 2 2 2 2 2 2

1 1
33 33
a b c 1  a  1  b  1  c  1  a  1  b 2 1  c 2
2 2 2 2 2 2 2

4  3 1  a  1  b  1  c  
2 2 2

1  a 1  b 1  c  
3
2

2  3 1  a 1  b 1  c   23  1  a 1  b 1  c 

Y esta última desigualdad es el problema 2.4.

Nota: En las soluciones oficiales el enunciado se reduce hasta la desigualdad de Nessbit (3.6).

8.6
Primera versión.
Observamos que no es posible que los tres números sean mayores que 1, pues
a, b, c  1  a 2  b2  c 2  abc  4 , contradiciendo la hipótesis del enunciado.

Supongamos que a  1 . Entonces 1  a  0 y por tanto


ab  bc  ca  abc  a(b  c)  bc(1  a)  0

Veamos ahora la desigualdad contraria.


Puesto que los tres números no pueden ser mayores que 1, podemos suponer, sin pérdida de
generalidad, b y c son ambos menores o ambos mayores que 1, es decir:
(b  1)(c  1)  0

Interpretando la igualdad a 2  b2  c2  abc  4 como una ecuación de segundo grado en a ,


tenemos que
 bc  (bc) 2  4(b 2  c 2  4)  bc  (4  b2 )(4  c 2 )
 
a 2 `(bc)a  b 2  c 2  4  0  a 
2

2

Y por tanto:
ab  bc  ca  abc  a(b  1)(c  1)  a  bc  a  bc .

Todo se reduce a demostrar que a  bc  2

 bc  (4  b2 )(4  c 2 )  bc  (4  b 2 )(4  c 2 )  2bc (4  b2 )(4  c 2 )  bc


a  bc   (*)
2 2 2

Y se demuestra aplicando la desigualdad Cauchy-Schwarz en la forma:


a1b1  a2b2  a2
1 
 a22 b12  b22 
A nuestro problema:
(4  b2 )(4  c 2 )  bc  (4  b2 ) (4  c 2 )  bc  4  b 2
 
 b2 4  c 2  c 2  16  4

Y por tanto
4
(*)   2 , tal y como queríamos ver.
2

Segunda versión.
La desigualdad inferior se demuestra igual que en la primera versión. Veamos la desigualdad
superior:
Igual que en la primera versión, podemos suponer sin pérdida de generalidad que b y c son
ambos mayores o iguales que 1 o ambos menores o iguales que 1, es decir, podemos suponer
que
(1  b)(1  c)  0

Es conocido que b2  c 2  2bc , y por tanto


4  a 2  b 2  c 2  abc  a 2  2bc  abc  a 2  bc(2  a) 
4  a 2  bc(2  a) 
2  a 2  a   bc(2  a) 
2  a  bc

Y por tanto:
ab  bc  ca  abc  ab  2  a  ac  abc  2  a1  bc  c  b  2  a(1  c)(1  b)  2

Fuente de estas dos versiones: https://artofproblemsolving.com/wiki/index.php/2001_USAMO_Problems/Problem_3

Nota: En dicha página web podemos encontrar una tercera versión mediante sustituciones
trigonométricas.

8.7
Primera versión.
Realizamos el siguiente cambio de variable:
x  a  b

y  b  c
z  a  c

Luego
x 2  (a  b) 2  a 2  b 2  2ab

y 2  (b  c) 2  b 2  c 2  2bc   x 2  y 2  z 2  2a 2  b 2  c 2  ab  bc  ac 
z 2  (a  c) 2  a 2  c 2  2ac 

Y por tanto:
4  a 2  b2  c 2  a  b  c   2a 2  b2  c2  ab  bc  ac   x2  y 2  z 2
2

Por otro lado,


x  z  y  2a
x  y  z  2b
y  z  x  2c

ab  1 bc  1 ca  1 4ab  4 4bc  4 4ca  4


  3    12 
a  b  b  c  c  a 
2 2 2
a  b 2 b  c 2 c  a 2
( x  z  y )( x  y  z )  4 ( x  y  z )( y  z  x)  4 ( y  z  x)( x  z  y )  4
   12 
x2 y2 z2
x 2  ( z  y ) 2  4 y 2  ( x  z ) 2  4 z 2  ( y  x) 2  4
   12 
x2 y2 z2
4  ( z  y)2 4  ( x  z)2 4  ( y  x) 2
1 1 1  12 
x2 y2 z2
4  ( z  y ) 2 4  ( x  z ) 2 4  ( y  x) 2
  9
x2 y2 z2

Por otro lado, 4  x2  y 2  z 2  4  x 2 , y 2 , z 2  x, y, z  2


4  z  y 
2
z, y  2  z  y  2  0  z  y   4  0  4  z  y   4 
4
 2
2 2
2
x x

y de la misma forma
4  ( x  z)2 4 4  ( y  x) 2 4
 2 ,  2
y 2
y z2 z

Luego
4  ( z  y ) 2 4  ( x  z ) 2 4  ( y  x) 2 4 4 4 1 1 1
2
 2
 2
 2  2  2  4 2  2  2   9
x y z x y z x y z 
1 1 1 9
Puesto que x 2  y 2  z 2  4  2
 2  2  , tal y como queríamos ver.
x y z 4

Esta última desigualdad es el problema 3.3b.

Segunda versión.
Se sigue el mismo desarrollo, hasta llegar a
x 2  ( z  y ) 2  4 y 2  ( x  z ) 2  4 z 2  ( y  x) 2  4
   12
x2 y2 z2

Puesto que, por hipótesis, 4  x 2  y 2  z 2 , seguimos así:


x 2  ( z  y ) 2  4 y 2  ( x  z ) 2  4 z 2  ( y  x) 2  4
  
x2 y2 z2
x 2  ( z  y ) 2  x 2  y 2  z 2 y 2  ( x  z ) 2  x 2  y 2  z 2 z 2  ( y  x) 2  x 2  y 2  z 2
  
x2 y2 z2
 
  
 
2 x 2  yz 2 y 2  xz 2 z 2  xy
 2

 x 2  yz y 2  xz z 2  xy 
  
x2 y2 z2  x
2
y2 z 2 
 x 2 y 2 z 2 yz xz xy   yz xz xy 
 2 2  2  2  2  2  2   2 3  2  2  2   12 
x y z x y z   x y z 
 yz xz xy  yz xz xy
 3  2  2  2   6  2  2  2  3
 x y z  x y z

Y esta última desigualdad es consecuencia directa de aplicar la desigualdad AM-GM:


yz xz xy yz xz xy
2
 2  2  33 2  2  2  33 1  3
x y z x y z

Tercera versión.
La condición a 2  b2  c 2  a  b  c   4 se puede reescribir como
2

a2  b2  c2  ab  bc  ac  2

Y por lo tanto:
2ab  2 2ab  a 2  b2  c 2  ab  bc  ac (a  b) 2  (c  a)(c  b) (c  a)(c  b)
  1
a  b2
a  b 2
a  b 2
a  b2
Y por tanto:
ab  1 bc  1 ca  1
  3
a  b  b  c  c  a 2
2 2

2ab  2 2bc  2 2ca  2


  6
a  b 2 b  c 2 c  a 2
(c  a)(c  b) (a  b)(a  c) (b  c)(b  a)
  3
a  b 2 b  c 2 c  a 2
Pero esta última desigualdad se demuestra aplicando la desigualdad AM-GM:
(c  a)(c  b) (a  b)(a  c) (b  c)(b  a) (c  a ) 2 (c  b ) 2 ( a  b ) 2
   3 3
a  b2 b  c 2 c  a 2 a  b2
3

Fuente de las versiones 2 y 3: https://artofproblemsolving.com/wiki/index.php?title=2011_USAMO_Problems/Problem_1&oldid=78098

8.8
Puesto que todos los términos son homogéneos, podemos suponer, sin pérdida de generalidad,
que a  b  c  3 , y por lo tanto la desigualdad a demostrar se transforma en la siguiente:
a  32  b  32  c  32  8
2a 2  3  a  2b 2  3  b  2c 2  3  c 
2 2 2

Observamos que
a  32  a 2  6a  9  1 a 2  2a  3  8a  6  1 1  8a  6   1  8a  6
 
2a 2  3  a  3a 2  6a  9 3 a 2  2a  3 3  a 2  2a  3  3 3a 2  6a  9
2

Y esto mismo podemos hacer en los otros dos términos de la suma, obteniendo
1 8a  6 1 8b  6 1 8c  6
 2   2   2 8
3 3a  6a  9 3 3b  6b  9 3 3c  6c  9
8a  6 8b  6 8c  6
 2  2 7
3a  6a  9 3b  6b  9 3c  6c  9
2

Ahora observamos que


8a  6 8a  6
3a 2  6a  9  3(a  1)2  6  6  
3a  6a  9
2
6

Y realizando esto mismo con los otros términos de la suma, llegamos a


8a  6 8b  6 8c  6
 2  2 
3a  6a  9 3b  6b  9 3c  6c  9
2

8a  6 8b  6 8c  6 1 8(a  b  c)  18 8  3  18
   8a  6  8b  6  8c  6   7
6 6 6 6 6 6

Fuente de la solución: https://artofproblemsolving.com/wiki/index.php?title=2003_USAMO_Problems/Problem_5


Nota: En esta página web se presentan tres soluciones alternativas más.

8.9
n
ak a1 a2 a3 a 1 1 1 1
k
k 1
2
 2
 2  2  ...  n2  a1  2   a2  2   a3  2   ...  an  2 
1 2 3 n 1  2  3  n 

Por un lado, tenemos el conjunto ordenado


1 1 1 1
2
 2  2  ...  2
1 2 3 n

Sea k1 , k2 ,..., kn  la permutación de 1, 2 , 3,..., n  tal que ak1  ak 2  ak3  ...  ak n

Entonces, por el Principio de Reordenación,


1 1 1 1 1 1 1 1
a1  2   a2  2   a3  2   ...  an  2   ak1  2   ak 2  2   ak 3  2   ...  ak n  2 
1  2  3  n  1  2  3  n 

Puesto que la expresión de la derecha es el reodenamiento mínimo.

Pero ak1  1 , ak 2  ak1  ak 2  2, ak3  3 ,..., ak n  n , y por tanto


1 1 1 1 1 1 1 1
ak1  2   ak 2  2   ak 3  2   ...  ak n  2   1  2   2  2   3  2   ...  n  2  
1  2  3  n  1  2  3  n 
1 1 1 1
    ... 
1 2 3 n

tal y como queríamos ver.

8.10
Primera versión.
0ba0 b  a
Aplicando la desigualdad AM-GM:
ab 
ab ab
2

2
 b  ab  b 
a b
2
 b a b 
ab
2
  
   
2 b a  b  a  b  2 b a  b  a  b  2 a b a  b  a a  b   
De nuevo aplicando la desigualdad AM-GM:
ab ab a  b  2a ab
ab     ab    ab   a   ab 
2 2 2 2

a b
2
 a  ab 
ab
2
 
 a a  b  a b  2 a a  b   

a  b  b  2 a b a  b 
Así pues, tenemos la desigualdad

(a  b) b  2 a b a  b  a a  b 
Que, después de unas transformaciones apropiadas, se convertirá en la desigualdad del
enunciado:
 
(a  b) b  2 a b a  b  a a  b  
ab
2 a
 a b
ab
2 b

a b
2

 2

ab a  b   a  b 2  a  b  
2 2
 
2

   a  b   
2 a 2 b 4a 4b
a  b  a  b  2 ab  a  b  a  b  a  b  2 ab  a  b2 
2 2 2

4a 4b 8a 2 8b
a  b  a  b  ab  a  b
2 2

8a 2 8b

Segunda versión.
a b
Observamos que b  a  b  a  a  b  2 a  1
2 a
a b
Y de la misma forma 1  , luego
2 b
2 2
a b a b  a b  a b
1    1 


 2 b 
2 a 2 b  2 a   


 a b 
2

1
 a b 
2


4a 4b
 a b 
2
a b  2

a b 
2 a b 
2
a b 
2


4a 4b
a  b 2   a b   2 a  b 2 
4a 4b
a  b   a  b  2 ab  a  b 2
2

8a 2 8b

Tercera versión.
ab
2

   ab
ab  2  ( a  b) 2
Observamos que  ab  
2 ab 2(a  b)  4 ab
 ab
2
Y por tanto la desigualdad del enunciado es equivalente a
4a  a  b  2 ab  4b , que es trivial puesto que a  b  0  a  ab  b

Fuente de la segunda y tercera versión: 101 Problems in Algebra from the training of the USA IMO team (Adreescu, Feng, 2001), página 29.

8.11
Haremos una demostración por reducción al absurdo. Supongamos que no se cumple la
desigualdad. Entonces:
(a  b)2  (a  b)2  4ab  4n  3  n2  1  n2  4n  2  (2n  1)2  a  b  2n  1

Y puesto que a, b son enteros, llegamos a a  b  2n .

Pero, por otro lado, por la desigualdad AM-GM,


 a  b   2n 
2 2

n  1  ab  
2
     n  n  1  n , absurdo.
2 2 2

 2    2
Así pues, se tiene que cumplir la desigualdad del enunciado, tal y como queríamos ver.

Veamos ahora la igualdad:

(a  b)2  (a  b)2  4ab  4n  3  n2  1  n2  4n  2  (2n  1)2  a  b  2n  1

a  b  4n  3  4n  3 es un cuadrado perfecto impar  4n  3  (2u  1)2 para cierto entero


no negativo u , y por tanto
4n  3  (2u  1)2  4u 2  4u  1  4n  4u 2  4u  4  n  u 2  u  1 , y a  b  2u  1 .
a  b  2n  1
  a  b  a  b  2n  1  2u  1  b  n  u  u 2  u  1  u  u 2  1
a  b  2u  1
a  b  2u  1  a  2u  1  b  2u  1  u 2  1  u 2  2u  2
Y por tanto:
    2
ab  u 2  2u  2 u 2  1  u 4  2u 3  3u 2  2u  2  u 2  u  1  1  n2  1

Luego la igualdad se cumple cuando:


a  u 2  2u  2 , b  u 2  1 , n  u 2  u  1 , a  b  2u  1 para todo entero no negativo u .
Fuente de esta solución: Solución oficial (SE)

8.12
Utilizaremos la desigualdad x  y  2( x  y) (ver Problema 3.3) y la igualdad acontece
si y solo si x  y .

a 2  b2  a 2  b2 
ab   2 ab    2ab  a 2  b 2  (a  b) 2  a  b
2  2 

La igualdad acontece cuando


a 2  b2
ab   2ab  a 2  b2  a 2  b2  2ab  0  (a  b)2  0  a  b  0  a  b
2

Nota: En las soluciones oficiales (SE) se presentan cuatro soluciones más.

8.13
Sea S  a  b  c .
a  b  3c a  3b  c 3a  b  c
  
3a  3b  2c 3a  2b  3c 2a  3b  3c
S  2c S  2b S  2a
  
3S  c 3S  b 3S  a
S  2c S  2b S  2a
 2 2 26
3S  c 3S  b 3S  a
S  2c  23S  c  S  2b  23S  b  S  2a  23S  a 
   6
3S  c 3S  b 3S  a
7S 7S 7S  1 1 1 
    6  7S      6  (*)
3S  c 3S  b 3S  a  3S  c 3S  b 3S  a 

Por otro lado,


3S  c  3S  b  3S  a  9S  S  8S  S 
1
3S  c  3S  b  3S  a 
8

Y por lo tanto
(*) 
7
3S  c  3S  b  3S  a  1  1  1   6  7  32  6  15
8  3S  c 3S  b 3S  a  8 8

En donde hemos utilizado la variación de la desigualdad AM-HM vista en 3.5a:


 
a1  ...  an  1  ...  1   n2
 a1 an 

Fuente de esta solución: Solución oficial (SE).

8.14
Primera versión.
Si b  0 , entonces a  1 , y por tanto
ax  by 2  ax2  by 2  x2  x2 , se cumple la igualdad. Lo mismo sucede cuando a  0 .
Supongamos que a, b  0 . Es una aplicación directa de la Desigualdad Cauchy-Schwarz "en la
forma de Engel" (4.3)

a 2 x 2 b 2 y 2 a x  b y   a x  b y  
2 2 2 2
ax  by 
2 2
   
  a  b   ax  by 2
a b a b  a b 

 ax by 
La igualdad acontece solo cuando las parejas  ,  y 
a, b son proporcionales, es 
 a b
decir, cuando
ax by ax by
   x y
a a b b a b

Segunda versión.
Podemos escribir
ax 2  by 2  ax  by   ax 2  by 2  a 2 x 2  b 2 y 2  2abxy 
2

 a(1  a) x 2  b(1  b) y 2  2abxy  abx 2  bay 2  2abxy  ab( x 2  y 2  2 xy ) 


 ab( x 2  y 2  2 xy )  ab( x  y ) 2

Que es claramente no negativa. La igualdad acontece cuando


ab( x  y)2  0 , es decir, cuando a  0 o b  0 o x  y .

Tercera versión.
En las soluciones oficiales se presenta una tercera versión mediante la Desigualdad de Jensen
aplicada a f ( x)  x 2 .

Fuentes de la segunda y tercera versión: Soluciones oficiales (SE pág. 1031)

8.15
a) es la desigualdad AM-GM.
b)
x2  y 2  x  y 

x  y   2 x 2  2 y 2  x  y 2  x 2  y 2  2 xy 
2 2

 
2  2  4
x 2  y 2  2 xy  0  ( x  y ) 2  0
c) Aplicando el apartado b) tenemos

2
 1 1
2 2  p q  2 2 2
 1  1  p q 1 1 1 1 pq 1 1 
 p     q   2  1     1    1  
 p  q  2  2 p q 2 pq  2 pq 
 
 

Luego el problema se reduce a demostrar que


pq
2
1 1 1 1 1
1 5  4  1  4 pq   pq     pq   pq   pq
pq pq 4 2 2 2
Que es precisamente el apartado a).

8.16
El problema equivale a demostrar que
P( x, y)  2 x 2 y  x 2  x  y  x3  xy 2  2 xy  0 si 0  x, y  1

Escribimos el polinomio como polinomio en la variable x:


P( x, y)   x3  (2 y  1) x 2  (1  2 y  y 2 ) x  y

Dividimos este polinomio por x  1 mediante el algoritmo de Euclides:

-1 2y 1 1  2 y  y2 y

1 -1 2y 1  y2
-1 2y 1  y2 1  y  y2

Así pues,
 
P( x, y )   x 2  2 yx  1  y 2 ( x  1)  1  y  y 2 
 
  x  2 yx  y ( x  1)  x  1  1  y  y 2 
2 2

 x 2

 2 yx  y (1  x)  x  1  1  y  y 2 
2

 ( x  y ) 2 (1  x)  x  y  y 2 
 ( x  y ) 2 (1  x)  x  y (1  y )

Y vemos que, efectivamente, para 0  x, y  1 los tres sumandos son positivos.

Fuente de la solución: Solución oficial (SE pág. 1022)

8.17
ab 2  a 2b  (1  a)(1  b) 2  (1  a) 2 (1  b)  ab(a  b)  (1  a)(1  b)2  (a  b)  
ab  2  ( a  b) 
 2ab   2(1  a)(1  b) 
 2   2 
ab  2  a  b  
 2 ab   2 (1  a)(1  b) 
 2   2 
 ab  a  b  
 2  ab   (1  a)(1  b)1  
  2   2  

Con lo que nuestro problema se reduce a demostrar que


ab  ab
0  a, b  1  ab   (1  a)(1  b)1   1
 2   2 

ab
Aquí utilizamos que 0  x  1  x  3 x , y que 0  a, b  1  0  1
2

Y por tanto, y aplicando la desigualdad AM-GM, tenemos


 a  b 3  a  b  1 a  b  1  3a  3b  a  b
ab   ab   a  b    
 2   2  3 2  3 2  2
 ab 3  a  b 1 ab
(1  a)(1  b)1    (1  a)(1  b)1    1  a  1  b  1  
 2   2  3 2 
 ab
 1  
 2 

Y finalmente:
ab  ab ab ab
ab   (1  a)(1  b)1   1 1
 2   2  2 2

8.18
a) Realizamos el siguiente cambio de variable:
x a y b z c
a x , b y , c z
x 1 a 1 y 1 b 1 z 1 c 1

La desigualdad a demostrar ahora es a 2  b2  c 2  1 , sujeta a la condición


a b c
1  xyz    
a 1 b 1 c 1
abc  (a  1)(b  1)(c  1)  abc  a  b  c  (ab  bc  ac)  1 
a  b  c  1  ab  bc  ac 
2(a  b  c  1)  (a  b  c) 2  (a 2  b 2  c 2 ) 
2(a  b  c)  2  (a  b  c) 2  (a 2  b 2  c 2 ) 
a 2  b 2  c 2  2  (a  b  c) 2  2(a  b  c) 
a 2  b 2  c 2  1  (a  b  c) 2  2(a  b  c)  1  (a  b  c  1) 2

De donde se deduce claramente que


0  (a  b  c  1)2  a 2  b2  c 2  1  a 2  b2  c2  1

tal y como queríamos ver.

b) Seguimos con el cambio de variable anterior. Queremos resolver la ecuación


a 2  b2  c 2  1

Y también
a 2  b2  c2  1  (a  b  c  1)2

Es decir, 0  a 2  b2  c2  1  (a  b  c  1)2  a  b  c  1
Luego tenemos que resolver el sistema
a 2  b 2  c 2  1

a  b  c  1

1  a 2  b 2  c 2  (a  b  c) 2  2(ab  bc  ac) 
1  12  2(ab  bc  ac) 
0  ab  bc  ac

a  b  c  1  c  1  a  b  0  ab  c(a  b)  ab  (1  a  b)(a  b) 
 a  a 2  b  ab  b 2  a(1  a)  b(1  a)  b 2  b 2  a(a  1)  b(a  1)  0

Entendiendo esta última expresión como una ecuación cuadrática en b, su discriminate es


  (a  1) 2  4  1  a(a  1)  (a  1) 2  4a(a  1)  (a  1)a  1  4a   (a  1) 3a  1 
 (1  a)(3a  1)

Para obtener valores racionales, es suficiente tomar valores de a racionales para los cuales
  (1  a)(3a  1) sea el cuadrado de un racional, de forma que el discriminante sea racional y
por tanto lo sea b y finalmente c).

k
Sea a  para ciertos enteros k y m .
m

 k  k   m  k  3k  m  (m  k )(3k  m)
  (1  a)(3a  1)  1   3  1    
 m  m   m  m  m2

Luego nuestro problema se reduce a encontrar enteros k, m para los cuales m  k y 3k  m


sean enteros.
Esto se obtiene tomando, por ejemplo, m  k 2  k  1 , para cualquier k entero, pues entonces
m  k  k 2  2k  1  (k  1)2
3k  m  3k  k 2  k  1  k 2  2k  1  (k  1)2

y ambos son cuadrados.

Así pues,
(k  1) 2 (k  1) 2 k 2 1
   
m2 m
(1  k / m)  (k 2  1) / m (m  k )  (k 2  1)
b 
2 2m

Tomando el valor de b con la suma:


(m  k )  (k 2  1) m  k  k 2  1 m  (k  k 2  1) 
b    m  m  2 2m  2 m  1
2m 2m 2m b   
 2 m 2 m m
m  2  k 2  k 1 2  k 2  k 1 

Y, finalmente
k m 1 k 1 1 k 1 k
a  b  c 1 c 1 a  b 1   1 1   
m m m m m m m

La condición a, b, c  1 descarta los casos k  0 y k  1 .

Aunque no es necesario, deshaciendo el cambio de variable llegamos a


k k 1
x , y  k  k2 , z  2
(k  1) 2
k

Fuente de la solución: Soluciones oficiales (SE , pág 964)

8.19

u
2  3t  t  1 2  3t  3t  1 2  3t  3t 
t t t
u
1 2  3t  3t
t
(*)
4 t
3 2  2 t 3 2  t 2
3 2t

Aplicando la desigualdad AM-GM:


2  3t  3t  12 2  3t  3t   22
t
t t

Luego
1 2t 1 1 1
(*)   u  2  (C )
3 22 2 3 2  3 12
t

La igualdad se alcanza cuando 2t  3t  3t  2t  6t , que es una ecuación que tiene solución


entre 0 y 1, pues aplicando Bolzano a la función continua f ( x)  2x  6 x :
t  0  f ( x )  20  6  0  1
t  1  f (1)  21  6 1  4

8.20
En primer lugar, aplicamos la desigualdad AM-GM ponderada (2.26):

aabbcc d d  aa  bb  cc  dd  a 2  b2  c2  d 2

Luego
a  2b  3c  4d  aabbccd d  a  2b  3c  4d a2  b2  c2  d 2 
Y por tanto es suficiente demostrar que

a  2b  3c  4d  a2  b2  c2  d 2  1 
Vemos que esta desigualdad no es homogénea y por lo tanto lo primero que hacemos es
homogeizarla, aprovechando la condición del enunciado: a  b  c  d  1

a  2b  3c  4d a2  b2  c2  d 2   a  b  c  d 3
Luego tenemos que demostrar que si a  b  c  d  0 , se cumple
a  2b  3c  4d a2  b2  c2  d 2   a  b  c  d 3
(ahora sin la restricción a  b  c  d  1 )

Primera versión (Solución oficial).


Vamos a utilizar las condiciones a  b  c  d  0 para ir "subiendo" poco a poco la parte
izquierda de la desigualdad:
a  2b  3c  4d  a  3b  3c  3d puesto que d  b
a  2b  3c  4d  3a  3b  3c  d puesto que d  a  2d  2a
a  2b  3c  4d  3a  b  3c  3d puesto que b  d  a  a  2a
a  2b  3c  4d  3a  3b  c  3d puesto que 2c  d  2a  b

Luego:
a  2b  3c  4d a 2  b 2  c 2  d 2  
a 2 a  2b  3c  4d  
b 2 a  2b  3c  4d  
c 2 a  2b  3c  4d  
d 2 a  2b  3c  4d  
a 2 a  3b  3c  3d  
b 2 3a  b  3c  3d  
c 2 3a  3b  c  3d  
d 2 3a  3b  3c  d   a  b  c  d   6abc  abd  acd  bcd   a  b  c  d 
3 3
1
La igualdad aparece cuando a  b  c  d  en cuyo caso tenemos:
4
a  2b  3c  4d a 2  b2  c 2  d 2   5  1
8
Fuente de esta versión: artoproblemsolving.com

Segunda versión ("Dumbassing", fuerza bruta).


Dado d  0 , podemos escribir c  d  c' , b  c  b'  b'c'd y a  b  a'  a'b'c'd , para
ciertos a' , b' , c'  0 , luego
a  2b  3c  4d  a'b'c'd  2(b'c'd )  3(c'd )  4d  a'3b'6c'10d
a  b  c  d  a'b'c'd  b'c'd  c'd  d  a'2b'3c'4d
a 2  b2  c2  d 2  (a'b'c'd )2  (b'c'd )2  (c'd )2  d 2

Lo que (quitando las tildes por comodidad) llegamos a la desigualdad


a  3b  6c  10d (a  b  c  d )2  (b  c  d )2  (c  d )2  d 2   a  2b  3c  4d 3
En donde ahora a, b, c, d  0 sin ninguna otra restricción más.

A falta de algo mejor, desarrollamos los dos lados. Primero la parte derecha:
a  2b  3c  4d 2  a 2  2ab  3ac  4ad  2ab  4b2  6bc  8bd  3ac 
 6bc  9c 2  12cd  4ad  8bd  12cd  16d 2 
 a 2  4b 2  9c 2  16d 2  4ab  6ac  8ad  12bc  16bd  24cd
a  2b  3c  4d 3 
a 2

 4b 2  9c 2  16d 2  4ab  6ac  8ad  12bc  16bd  24cd a  2b  3c  4d  
 a 3  4ab 2  9ac 2  16ad 2  4a 2b  6a 2c  8a 2 d  12abc  16abd  24acd 
 2a 2b  8b3  18bc 2  32bd 2  8ab 2  12abc  16abd  24b 2c  32b 2 d  48bcd 
 3a 2c  12b 2c  27c 3  48cd 2  12abc  18ac 2  24acd  36bc 2  48bcd  72c 2 d 
 4a 2 d  16b 2 d  36c 2 d  64d 3  26abd  24acd  32ad 2  48bcd  64bd 2  96cd 2 
 a 3  12ab 2  27ac 2  48ad 2  6a 2b  9a 2c  12a 2 d  36abc  48abd  72acd 
 8b3  54bc 2  96bd 2  36b 2c  48b 2 d  144bcd  27c 3  144cd 2  108c 2 d  64d 3

Y ahora desarrollamos la parte izquierda:


(a  b  c  d ) 2  a 2  ab  ac  ad  ab  b 2  bc  bd  ac  bc  c 2  cd 
 ad  bd  cd  d 2  a 2  b 2  c 2  d 2  2ab  2ac  2ad  2bc  2bd  2cd
(b  c  d ) 2  b 2  bc  bd  bc  c 2  cd  bd  cd  d 2 
b 2  c 2  d 2  2bc  2bd  2cd

(c  d )2  c 2  cd  cd  d 2  c 2  2cd  d 2
a  b  c  d 2  (b  c  d )2  (c  d )2  d 2 
 a 2  b 2  c 2  d 2  2ab  2ac  2ad  2bc  2bd  2cd  b 2  c 2  d 2  2bc  2bd  2cd 
 c 2  2cd  d 2  d 2  a 2  2b 2  3c 2  4d 2  2ab  2ac  2ad  4bc  4bd  6cd
a  3b  6c  10d (a  b  c  d )2  (b  c  d )2  (c  d )2  d 2  
 a  3b  6c  10d a 2  2b 2  3c 2  4d 2  2ab  2ac  2ad  4bc  4bd  6cd  
 a 3  2ab 2  3ac 2  4ad 2  2a 2b  2a 2c  2a 2 d  4abc  4abd  6acd 
 3a 2b  6b3  9bc 2  13bd 2  6ab 2  6abc  6abd  12b 2c  12b 2 d  18bcd 
 6a 2c  12b 2c  18c 3  24cd 2  12abc  12ac 2  12acd  24bc 2  24bcd  36c 2 d 
 10a 2 d  20b 2 d  30c 2 d  40d 3  20abd  20acd  20ad 2  40bcd  40bd 2  60cd 2 
 a 3  8ab 2  15ac 2  24ad 2  5a 2b  8a 2c  12a 2 d  22abc  30abd  38acd 
 6b3  33bc 2  52bd 2  24b 2c  32b 2 d  82bcd  18c3  84cd 2  82bcd  66c 2 d  40d 3

Y vemos que los coeficientes de los monomios de la parte derecha son todos mayores que los
de la izquierda, lo que garantiza que se cumple la desigualdad.

8.21
El valor máximo se obtiene como aplicación directa de la desigualdad AM-GM con los valores
a 2 , b2 , c 2 , d 2 :
12 a 2  b 2  c 2  d 2 4 2 2 2 2
3   a b c d  abcd  9  abcd  abcd
4 4
Y este valor lo podemos obtener tomando, por ejemplo: a  b  3 , c  d   3 .

Veamos ahora el valor mínimo.


Tomando a  b  c  1 , d  3 se cumplen las condiciones del enunciado y abcd  3 .
Veamos que este valor es mínimo.
Para que el producto de cuatro números sea negativo, uno o tres de dichos números han de ser
negativos. Supongamos que a, b, c  0 y d  (a  b  c)  0 .

Aplicamos la desigualdad AM-GM a los valores a, b, c  0 :


abc 3 abc d  d
3 3 3

 abc  abc      
3  3   3  27
Aplicando la desigualdad AM-QM a estos mismos valores:

abc a 2  b2  c2 12  d 2 d 12  d 2 d 2 12  d 2
     
3 3 3 3 3 9 3
 2

 d  3 12  d  36  3d  4d  36  d  9  3  d  0
2 2 2 2

Por lo tanto:
 d3 
abc    d 3 (3) d 3  27
27   abcd     3
27 9 9
 3  d  0

Con igualdad si y solo si a  b  c , y en este caso


abcd 0  3a  d  0  d  3a 
 2  2

a  b  c  d  12 3a  d  12 12  3a  (3a)  3a  9a  12a 
2 2 2 2 2 2 2 2 2

a  b  c  1, d  3  abcd  3
Que es la solución obtenida al principio.
Fuente: Soluciones oficiales (OME, página 890)

9.1
Vemos que la desigualdad se adapta al modelo de la desigualdad Cauchy-Schwarz:

n 2
 
 m2 p 2  q 2  np  mq
2

a b
Tomando n  p  1 , m  y q , que están bien definidos pues
sin x cos x

0 x  0  sin x, cos x  1
2

2 2
 a  a   a b   ab 
1   1    1  1    1   
 sin x   cos x   sin x cos x   sin x cos x 
2
 1 
 1  ab  

 sin x cos x 
1
Solo queda demostrar que 2
sin x cos x

En efecto,
1 sin 2 x  cos 2 x sin 2 x cos 2 x
   
sin x cos x sin x cos x sin x cos x sin x cos x
sin x cos x
  2
cos x sin x

Por la desigualdad del problema 1.2

9.2
9 x 2 sin 2 x  4 9 x 2 sin 2 x 4 4
f ( x)     9 x sin x 
x sin x x sin x x sin x x sin x
4
Sean u  9 x sin x , v 
x sin x
4
Observamos que su producto es constante: u v  9 x sin x  36
x sin x

Aplicamos la desigualdad AM-GM:


uv
 uv  36  6  u  v  12
2
Luego el valor mínimo de la función es 12.
4 4
La igualdad se obtiene si u  v  9 x sin x   x 2 sin 2 x 
x sin x 9
   
2
4 4
La función g ( x)  x sin x cumple g (0)  0 y g       , luego el valor
2 2
se
2 2 9 9

alcanzará entre 0 y .
2

9.3
 
Vemos que las secuencias sin 3 x, cos 3 x y 
 1
,
1 
 están ordenadas en el mismo orden:
 cos x sin x 
 1 1
0  x   sin 3 x1  sin 3 x2  sin x1  sin x2  cos x1  cos x2  
2 cos x1 cos x2

Luego , por el Principio de la Reordenación (ver Tema 5):


sin 3 x cos 3 x  sin 3 x cos 3 x 
f ( x)   1 1  1 1   sin 2 x  cos 2 x  1
 cos x sin x   sin x cos x 

9.4
Primera versión. Aplicando la desigualdad Cauchy-Schwarz.
Vemos que se trata de una aplicación directa de la desigualdad Cauchy-Schwarz tomando
a b
a1  b1  1 y a2  y b2 
sin x cos x
2
 a b   a  b 
1  1    1  1  
 sin x cos x   sin x  cos x 
 
2
 ab   a  b 
1  1    1   1  
 sin x cos x   sin x  cos x 

1
Solo queda demostrar que sin x cos x  , lo cual se puede demostrar como aplicación de la
2
desigualdad GM-QM:
sin 2 x  cos 2 x 1 1
sin x cos x    sin x cos x 
2 2 2

O también se podría haber demostrado directamente mediante la identidad del seno del ángulo
doble:
1 1 1
sin x cos x  sin 2 x  1 
2 2 2

Luego
1 ab ab
sin x cos x    2ab   2ab
2 sin x cos x sin x cos x
2

 1
ab
sin x cos x

 1  2ab  1 
ab 

  1  2ab
sin x cos x 
2

Con lo que llegamos finalmente a la desigualdad del enunciado.

Segunda versión. Aplicando la desigualdad AM-GM.

Desarrollamos los dos lados de la desigualdad:

1  2
 
2ab  1 
a 
 1 
b 

 sin x   cos x 
b a ab
1  2 2ab  2ab  1    
cos x sin x sin x cos x
b a ab
2 2ab  2ab    (*)
cos x sin x sin x cos x

Pero, aplicando la desigualdad AM-GM,


ab b a
2  
sin x cos x cos x sin x

1
y puesto que sin x cos x (ver la primera versión)
2
1 ab ab
sin x cos x     2ab
2 sin x cos x 1/ 2
b a
Y por tanto 2 2ab  
cos x sin x

1 ab
Por otro lado, está claro que sin x cos x 
 2ab  , y sumando estas dos
2 sin x cos x
desigualdades llegamos a demostrar la desigualdad (*) .

Fuente de esta segunda versión: 103 Trigonometry Problems From the Training of the USA IMO Team (Titu Andreescu, 2005) , pág. 93.

10.5
Puesto que son los lados de un triángulo , podemos escribir
a  y  z , b  z  x , c  x  y , con x, y, z  0

Luego
a 2  b 2  c 2  ( y  z ) 2  ( z  x) 2  ( x  y ) 2 
 y 2  2 yz  z 2  z 2  2 zx  x 2  x 2  2 xy  y 2 

 2 y 2  2 yz  2 z 2  2 zx  2 x 2  2 xy  2 x 2  y 2  z 2  xy  yz  xz  
 2 2
 
2 a  b  c  4 x  y  z  xy  yz  xz
2 2 2 2

a  b  c 2   y  z  z  x  x  y 2  2 x  2 y  2 z 2  4x  y  z 2 

 4 x 2  xy  xz  yx  y 2  yz  xz  yz  z 2  
 4x 2
 y  z  2xy  yz  xz 
2 2

Luego

a  b  c 2  2 a 2  b2  c 2  
  
4 x 2  y 2  z 2  2xy  yz  xz   4 x 2  y 2  z 2  xy  yz  xz  
4xy  yz  xz   0

10.6
Puesto que son los lados de un triángulo , podemos escribir
a  y  z , b  z  x , c  x  y , con x, y, z  0

ab  bc  ac  x2  y 2  z 2  3( xy  yz  xz )  p  3q
a 2  b2  c2  2x2  y 2  z 2  xy  yz  xz   2 p  q   2 p  2q
2ab  bc  ac   2 p  6q

Está claro que a 2  b2  c 2  2ab  bc  ac  pues 2 p  2q  2 p  6q

Veamos la primera desigualdad:


a 2  b2  c 2  ab  bc  ac   2 p  2q  p  3q  p  q  0  p  q 
a 2  b2  c 2  ab  bc  ac
Y esta desigualdad se demostró en el bloque teórico.

10.7
a) Aplicando la Transformación de Ravi:
a x y, b y z, c zx
a x y x y
  y de la misma manera
bca y z z xx y 2z
b yz c zx
 , 
c ab 2x abc 2y

a b c x  y y  z z  x 1 x  y y  z z  x 
         
bca cab a bc 2z 2x 2y 2 z x y 
1 x y y z z x 1 x z y z y x 1
                 (2  2  2)  3
2 z z x x y y  2  z x z y x y  2

b) De nuevo, mediante la Transformación de Ravi:


bc a c a b a bc 2z 2x 2y  z x y 
      2     3
a b c x y yz zx  x y y z z x
Es la Desigualdad de Nesbitt (3.4).

10.8
Ver problema PG/6.15

10.9
Ver problema PG/6.59

10.10
Ver problema PG/7.7

10.11
Ver problema PG/7.8

10.12
Ver Problema PG/7.9

10.13
Podemos suponer, sin pérdida de generalidad, que a  b  c .

Entonces, puesto que estamos trabajando con los lados de un triángulo, se cumple:

ab  c  a   ba  c  b  ca  b  c 

En efecto, veamos la segunda desigualdad:


ba  c  b   ca  b  c   ab  bc  b 2  ac  bc  c 2 
ab  b2  ac  c 2  0  b 2  c 2  ac  ab  (b  c)(b  c)  a(b  c)  (b  c)b  c  a 

y, efectivamente, b  c pues suponemos b  c y b  c  a  0 por la desigualdad triangular.


Una vez establecido esto, el problema se resuelve mediante el método de Reordenación,
1 1 1
teniendo en cuenta que a  b  c   
a b c

ab  c  a   ba  c  b  ca  b  c   ab  c  a   ba  c  b   ca  b  c 


1 1 1 1 1 1
a b c c a b

Simplificamos la parte de la izquierda:


ab  c  a   ba  c  b   ca  b  c   b  c  a  a  c  b  a  b  c  a  b  c
1 1 1
a b c

Simplificamos la parte de la derecha:


ab  a 2 bc  b2 ac  c 2
ab  c  a   ba  c  b   ca  b  c  
1 1 1
a b c
c a b c a b

Cancelando términos llegamos a:


ab  a 2 bc  b 2 ac  c 2 a(b  a) b(c  b) c(a  c)
0      
c a b c a b
a(a  b) b(b  c) c(c  a)
0  
c a b

Multiplicando por abc a ambos lados llegamos a la desigualdad del enunciado:


0  a 2b(a  b)  b2c(b  c)  ac2 (c  a)

10.14
La desigualdad de la izquierda:
3(ab  bc  ca)  (a  b  c)2
está demostrado en el Problema 2.3b. No hace falta que sean lados de un triángulo.

Veamos la desigualdad de la derecha.

Aplicando la Transformación de Ravi,


a  b  c  2( x  y  z)  (a  b  c)2  4( x  y  z)2
ab  bc  ca  x 2  y 2  z 2  3( xy  yz  xz )

Luego
(a  b  c) 2  4(ab  bc  ca )  4( x  y  z ) 2  4(ab  bc  ca ) 
( x  y  z ) 2  x 2  y 2  z 2  3( xy  yz  xz ) 
x 2  y 2  z 2  2( xy  yz  xz )  x 2  y 2  z 2  3( xy  yz  xz ) 
2( xy  yz  xz )  3( xy  yz  xz ) 
0  xy  yz  xz , lo cual es cierto.

10.15
La desigualdad de la izquierda es el problema 2.5a.
Para la desigualdad de la derecha, basta ver que:
2(ab  bc  ca)  ab  bc  ac  ab  bc  ac  a(b  c)  b(a  c)  c(a  b)
Y tener en cuenta que b  c  a , a  c  b y a  b  c .

10.16
La desigualdad de la izquierda es la desigualdad de Nesbitt (3.6)
Veamos la desigualdad de la derecha. Observamos que
abc
2a  b   a  b  a  b  a  b  c  a  b 
c 2c
 
2 ab abc

b 2b c 2c
Y de la misma manera:  y 
cb abc ab abc
Por lo tanto
a b c 2a 2b 2c 2(a  b  c)
      2
bc ca ab abc abc abc abc

10.17
Basta tener en cuenta que
     
a b2  c 2  a 2  b c2  a 2  b2  c a 2  b2  c2  a 2 (b  c  a)  b2 (c  a  b)  c2 (a  b  c)

y aplicar el problema 10.10

10.18
a  b b  c c  a (a  b)(a  c)(b  c)
Utilizaremos la siguiente identidad:   
a  b b  c c  a (a  b)(b  c)(a  c)
En efecto:
(a  b)(b  c)(c  a)  a 2b  ab 2  a 2c  b 2c  ac 2  bc 2
(b  c)(a  b)(a  c)  a 2b  ab 2  a 2c  b 2c  ac 2  bc 2
(c  a)(a  b)(b  c)  b 2c  ac 2  bc 2  a 2b  ab 2  a 2c

Y por tanto el numerador de la suma de las tres fracciones es:


a 2b  ac2  bc 2  b2c  ab2  a 2c  (a  b)(a  c)(b  c)

Luego
a  b b  c c  a (a  b)(a  c)(b  c) (a  b)(a  c)(b  c)
     (*)
a  b b  c c  a (a  b)(b  c)(a  c) (a  b)(b  c)(a  c)

Ahora utilizamos el Problema 2.3


(a  b)(b  c)(a  c)  8abc

Y por tanto
(a  b)(a  c)(b  c) 1 a  b a  c b  c
(*)    (**)
8abc 8 c b a

Y la equivalencia b de la desigualdad triangular:


bc ac a b
a  bc  1 , b  a  c  1 , c  a  b  1
a b c
Luego, finalmente:
1 1
(**)   1  1  1 
8 8

10.19
Se deduce directamente del problema 10.14:
3(ab  bc  ca )  (a  b  c)2  4(ab  bc  ca ) 
32  (a  b  c)2  4  3  3  a  b  c  2 3

10.20
a)
abc  a  b  c 
( s  a)( s  b)  ab    a   b   ab 
 2  2 
a  b  c  2a a  b  c  2b   4ab  b  c  a a  c  b  4ab
Pero
4ab  b  c  a a  c  b  4ab  c2  2ab  a 2  b2  (a  b)2  c2  0
Pues a  b  c  a  b  c 2
2

Observación: Una manera alternativa:


( s  a)( s  b)  ab  s 2  sb  as  ab  ab  s 2  sb  as  0 
s 2  s ( a  b)  0  s ( a  b)  s 2 
a  b  s  2(a  b)  a  b  c  2a  2b  a  b  c  a  b  c
Lo cual es cierto por la desigualdad triangular.

b) Multiplicando la parte izquierda por 4:


(2s  2a)(2s  2b)  (2s  2b)(2s  2c)  (2s  2c)(2s  2a) 
 (b  c  a)(a  c  b)  (a  c  b)(a  b  c)  (a  b  c)(b  c  a) 
 
 2ab  ac  bc   a 2  b 2  c 2  2ab  ac  bc   ab  ac  bc   ab  ac  bc

Donde hemos utilizado a2  b2  c2  ab  ac  bc demostrado en el problema 10.15.

10.21
Ver problema PG/7.67

11.1
Ver problema PG/6.58

11.2
Ver problema PG/6.72

11.3
Ver problema PG/6.74

11.4
Ver problema PG/6.80

11.5
Ver problema PG/6.85

11.6
Ver problema PG/6.84

11.7
Sean P, Q y R los baricentros respectivos de ABC , BCD y ACD .
En este problema utilizaremos la siguiente notación: Denotaremos por M AB el punto medio del
segmento AB .

En primer lugar vamos a demostrar que si PQR es equilátero entonces ABC es equilátero.

Veamos que PR es paralelo a BD.


1 CM BC CM CD
   CM BC M CD  CBD  M BC M CD // BD
2 BC CD

Sabemos (ver GA/11.5.3) que las medianas de un triángulo se cortan en razón 2:1. En
particular:
2 AP AR
   APR  AM BC M CD  PR // M BC M CD
3 AM BC AM CD

Y por tanto PR // M BC M CD // BD  PR // BD

Veamos que QR es paralelo a AB.


Basta aplicar de nuevo GA/11.5.3:
2 AR BQ
   BM CD A  QM CD R  QR // AB
3 AM CD BM CD

Finalmente, veamos que PQ es paralelo a AD.


De nuevo por 11.5.3,
2 CP CQ
   M AB CM BD  PCQ  PQ // M AB M BD
3 CM AB CM BD
Pero por otro lado,
1 BM AB BM BD
   ABD  M AB BM BD  AD // M AB M BD
2 BA BD

y por tanto AD // M AB M BD // PQ  AD // PQ

Así pues, ABC es también un triángulo equilátero.

Queremos calcular el área máxima de un cuadrilátero ABCD con BC  2 y CD  6 , y en


donde ABC es equilátero. Sea   BCD .

El área del triángulo BCD será BCD   2  6  sin   6 sin  .


1
2
Por otro lado, aplicando el Teorema del Coseno,
BD 2  22  62  2  2  6  cos   40  24 cos  .

El área del triángulo ABD será

ABD   3 BD 2  3 40  24 cos    310  6 cos  


4 4
Luego la función que tenemos que hacer máxima es
ABCD   BCD   ABD   6 sin   310  6 cos    6 sin   6 3 cos   10 3 
 
 6 sin   3 cos   10 3

Para encontrar el máximo de esta expresión realizamos la siguiente substitución trigonométrica:


1 3  1 3 
sin   3 cos   2 sin   cos    2 sin   cos   
2 2  2 2 
2cos60º sin   sin 60º cos    2 sin(  60º )

En donde hemos aplicado la identidad del seno de la diferencia de ángulos (ver TR/5.1),

El valor máximo de 2 sin(  60º ) se obtendrá cuando:


sin(  60º )  1    60º  90º    90º60º  150º

Para este valor, el área obtenida es


1   3 
ABCD   6sin   
3 cos   10 3  6  3
2
   10 3 

  2 
 12  10 3 (C )

Observación. Si no queremos utilizar trigonometría, podemos determinar el área mediante la


fórmula de Heron (GA/10.5.11). Sea x la longitud del triángulo ABD .
x  26 x 8
p 
2 2

 x  8  x  8  x  8  x  8 
BCD      2   6   x 
 2  2  2  2 
 x  8  x  4  x  4  8  x  1
      x  8x  4x  48  x  
 2  2  2  2  4


1
4
 
82  x 2 x 2  4 2 
1
4
  
64  x 2 x 2  16 
1
4

 x 4  80 x 2  1024

Luego la fórmula del área total será:


ABCD   BCD   ABD   1  x 4  80 x 2  1024  3x 2 
4


1
4
 2
576  x 2  40 
4
3 2
x 

Haciendo ahora la substitución k  x 2  40 , llegamos a la expresión


1
576  k 2 
3
k  40  1 576  k 2  3 k  10 3
4 4 4 4

Podemos ignorar el término 10 3 para concentrarnos en encontrar el máximo de


1
4

576  k 2  3k 
Aplicando la desigualdad de Cauchy-Schwarz:
 2


 2


  
576  k 2  3k  12  3  576  k 2   k  
2
2

 
 4 576  k 2  k 2  4  576  2304  482  576  k 2  3k  48 
1
4

576  k 2  3k 
48
4
 12 
Y finalmente
ABCD   12  10 3 (C)
Fuente de estas soluciones: Soluciones oficiales en artofproblemsolving.com

12.5
a)
Aplicando La Desigualdad Triangular al triángulo ABD , c  AD  a / 2
Aplicando La Desigualdad Triangular al triángulo ACD , b  AD  a / 2
Luego, sumando las dos desigualdades anteriores,
bca
b  c  2 AD  a  b  c  a  2 AD   AD
2

b) Sea 1  BAD ,  2  DAC ,   ABD ,   ACD

1
Comparando lados y ángulos en ABD : AD  a  BD  1  
2

1
Comparando lados y ángulos en ACD : AD  a  CD   2  
2
Luego 180º  1   2      1   2  1   2  2BAD   BAD 
180º
 90º .
2

12.6 Ver problema PG/7.6

12.7
Aplicamos la Desigualdad de Leibniz 12.4:

9R 2  a 2  b2  c 2

Y aplicando 4RABC   abc (ver GA/11.6.5)

Tenemos:
a 2b 2 c 2
4 RABC   abc  16 R ABC  a b c R  
2 2 2 2 2 2

16ABC 
2

9 a 2b 2 c 2 a 2b 2 c 2 a 2  b2  c2 a 2  b2  c2 a 2b 2 c 2
 a b c 
2 2 2
  
16ABC  16ABC  16ABC 
2 2 2
9 9
a 2  b2  c2
 4ABC  
abc 3abc
 
3 4ABC  a 2  b2  c2
Y ahora, aplicando la desigualdad Cauchy-Schwarz, tenemos

a  b  c  3 a 2  b2  c 2

Y por tanto

4ABC    4 3ABC  
3abc 3 3abc 3 3abc 9abc
 
a b c
2 2 2
3 a b c
2 2 2 abc abc

Tal y como queríamos ver.

12.8
Aplicamos la Desigualdad Cauchy-Schwarz (4.1) a las ternas
  
ap , bq , cr y 1/ a ,1/ b ,1/ c 
para obtener
1 1 1
p  q  r  ap  bq  cr    (*)
a b c

Interpretando geométricamente los valores p, q, r como alturas de los tres triángulos interiores
que determina P, la expresión ap  bq  cr es dos veces su área, y aplicando GA/11.13.1
tenemos:
ap  bq  cr  2ABC   2
abc abc

4R 2R
Así pues,
abc bc  ac  ab abc  bc  ac  ab  bc  ac  ab
(*)    
2R abc 2R  abc  2R

Y por último solo nos queda aplicar el Problema 2.5a


bc  ac  ab  a 2  b2  c2

para llegar al resultado deseado.

Fuente de esta solución: web "El blog de Leo" blog.nekomath.com

12.9
a) El punto P divide el triángulo en tres triángulos de alturas PD, PE, y PF, con bases todas
iguales al lado a , luego
ABC   aPD  aPE  aPF  a PD  PE  PF 
2 2 2 2
Pero, por otro lado, ABC  
ah
2
De donde se deduce directamente que PD  PE  PF  h

b) Utilizamos 4.4 y el Lema de Viviani demostrado en el apartado a:


PD  PE  PF  1  1  1   9  1  1  1  9 9
  (*)
 PD PE PF  PD PE PF PD  PE  PF h
2
 a  3a
2
3a
Por otro lado, por Pitágoras, h 2  a 2     h
2 4 2
9 6 3
(*)  
3a / 2 a

Tal y como queríamos ver.

b) De nuevo, aplicando 4.4,


PD  PE  PE  PF  PF  PD  1

1

1 
9
 PD  PE PE  PF PF  PD 
 1 1 1 
2h    9
 PD  PE PE  PF PF  PD 
 1 1 1  9 9 3 3
     
 PD  PE PE  PF PF  PD  2h 2 3a / 2 a

12.10
a)
ABC   a  ha  2ABC   ha 
1

a
2 a ha 2ABC 
1 1 1
  
a

b

c

1
a  b  c   s  1 
ha hb hc 2ABC  2ABC  2ABC  2ABC  ABC  r
rs  ABC 
Lo cual es cierto por GA/11.4.8.

Observación: Una manera más elegante podría ser tener en cuenta que, si I es el incentro del
triángulo:
r

ra

IBC  , y de la misma forma r  ICA y r  IAB 
ha ha  a ABC  hb ABC  hc ABC 

Y por tanto
r r r
  
IBC   ICA  IAB   ABC   1
ha hb hc ABC  ABC  ABC  ABC 

b) Aplicamos 4.4 y el apartado a:


 
ha  hb  hc  1  1  1   32  9 
 ha hb hc 
 r r 
ha  hb  hc  r
    9r  ha  hb  hc  9r
 ha hb hc 

12.11
a) Aplicaremos que las áreas de triágulos con la misma base son proporcionales a las alturas
correspondientes. En nuestro caso:
AD BE CF ABC  ABC  ABC 
     
HD HE HF BHC  AHC  AHB
 1 
 ABC 
1 1
  
 BHC  AHC  AHB 
 1  2
 BHC   AHC   AHB
1 1
  3 9
 BHC  AHC  AHB 

En donde hemos aplicado la desigualdad 4.4.

b) Sean S  ABC  , S1  HBC  , S2  HAC  , S3  HAB


HD HD S1 S1 HE S2 HF S3
   , y de la misma forma  y 
HA DA  HD S  S1 S2  S3 HB S1  S3 HC S1  S2

y por tanto:
HD HE HF S1 S2 S3 3
     
HA HB HC S2  S3 S1  S3 S1  S2 2

Es una aplicación directa de la Desigualdad de Nesbitt (3.6).

Fuente de la solución: Inequalities A Mathematical Olympiad Approach (Radmila Bulajich Manfrino) pág. 62.

12.12
Sea a  AC , b  CE y c  AE .

Aplicamos la Desigualdad de Ptolomeo 12.2 al cuadrilátero ACEF :


a  FE  b  AF  FC  c
Puesto que se cumple FA  FE , tenemos
c FA
FA(a  b)  FC  c  
a  b FC
Y de la misma manera:
a BC b DE
 y 
b  c BE c  a DA

Y por tanto:
BC DE FA a b c 3
     
BE DA FC b  c c  a a  b 2

Que es la desigualdad de Nesbitt (3.6).

12.13
En primer lugar vamos a estudiar uno de los segmentos por separado.
Consideremos el segmento AP.
Sea A' el punto medio del lado BC y P' el punto medio del arco BC.
Sean D y D' las respectivas proyecciones de A y P en el segmento BC.

AL AD AD
Por semejanza de triángulos,  
LP D' P A' P'
Luego el valor mínimo se obtiene cuando P  P' , es decir, cuando AP es la bisectriz del ángulo
A (ver GA/11.12.1).
Así pues, a partir de ahora supondremos que AP, BQ y CR son las bisectrices respectivas de los
ángulos A, B y C.

Pero en este caso sabemos (ver GA/11.4.4) que


ac ab   a 2 
BL  , LC  y AL2  bc1   
bc bc   b  c  
 

Y por tanto:
  a 2    a 2 

bc 1     bc1  
  
2 2   b  c   b  c   b  c 2  a 2
   
AL AL AL
  
LP AL  LP BL  LC ac

ab a 2
bc a2
bc bc b  c 2
Y de la misma manera, con los otros dos segmentos del enunciado tenemos:
BM (c  a) 2  b 2 CN (a  b)2  c 2
 , 
MQ b2 NR c2

Y por tanto, aplicado la desigualdad del Problema 2.5b:


AL BM CN b  c   a 2 (c  a) 2  b 2 (a  b) 2  c 2
2
     
LP MQ NR a2 b2 c2


b  c  (c  a ) 2 ( a  b ) 2
2
  3
1bc ca ab
 
2

3
  (*)
a2 b2 c2 3 a b c 

Y finalmente, teniendo en cuenta que


bc ca ab b c c a a b b a c b a c
              2226
a b c a a b b c c a b b c c a

Llegamos a
1
(*)  62  3  9
3

Observamos que la igualdad se cumple si y solo si a  b  c .

Observación: Otra manera de acabar este problema sería la siguiente:


b  c 2  (c  a)2  (a  b)2  3 
a2 b2 c2
 a 2 b 2   b 2 c 2   c 2 a 2   ab bc ca 
  2  2    2  2    2  2   2 2  2  2   3  2  2  2  2  3  3  9
b a  c b  a c  c a b 

En donde hemos tenido en cuenta que


a 2 b2 ab bc ca abbcca
2
 2  2 y 2  2  2  33 2 2 2  3
b a c a b abc

Fuente de esta solución: Inequalities A Mathematical Olympiad Approach (Radmila Bulajich Manfrino), pág. 63

12.14
Nos basamos en que las áreas de triángulo compartiendo una misma base son proporcionales a
las alturas:

AD BE CF ABC  ABC  ABC 


     
PQ PR PS BPC  CPA APB 
 1 1 
 ABC  
1
  
 BPC  CPA APB  
 1 1  2
 BPC   CPA  APB  
1
  3 9
 BPC  CPA APB  

En donde hemos utilizado que ABC   BPC   CPA  APB  y la desigualdad 4.4.

12.15
Si P es el baricentro del triángulo, basta aplicar GA/11.5.3b:
AP BP CP
   2226
PL PM PN
AP BP CP
Veamos el recíproco. Supongamos que    6.
PL PM PN
Entonces:
AP BP CP AP PL BP PM CP PN
  69      
PL PM PN PL PL PM PM PN PN
AP  PL BP  PM CP  PN AL BM CN
     
PL PM PN PL PM PN

PL BM PN
Pero sabemos, por GA/9.2.3, que   1
AL AM CN
Y por la desigualdad 4.4 que
 PL BM PN  AL BM CN 
      9
 AL AM CN  PL PM PN 

Por lo anterior, esta desigualdad es igualdad, y esto solo sucede cuando


AL BM CN
  3
PL PM PN

Y esto solo sucede cuando P es el baricentro (faltaría por demostrar esto último).

12.16
a) Sea H el ortocentro del triángulo. Por GA/11.6.9 sabemos que HD=DD', HE=EE', HF=FF',
luego
AD BE CF AD BE CF
    
DD' EE ' FF ' HD HE HF

Por GA/9.2.3 sabemos que


HD HE HF
  1
AD BE CF

Aplicando la Desigualdad 4.4 tenemos


 HD HE HF  AD BE CF 
      9
 AD BE CF  HD HE HF 

AD BE CF
Luego    9 , tal y como queríamos ver.
HD HE HF

b)
AD ' BE ' CF ' AD  DD' BE  EE ' CF  FF '
     
AD BE CF AD BE CF
DD' EE ' FF ' DD' EE ' FF '
1 1 1  3   4
AD BE CF AD BE CF

DD' EE ' FF '


Puesto que ya hemos visto en el apartado anterior que    1.
AD BE CF
Aplicando la Desigualdad 4.4 tenemos
 AD ' BE ' CF '  AD BE CF 
      9
 AD BE CF  AD ' BE ' CF ' 
 AD BE CF   AD BE CF  9
 4   9   
 AD ' BE ' CF '   AD ' BE ' CF '  4
tal y como queríamos ver.

12.17
a) Por GA/11.4.4d sabemos que
4bcs ( s  a) 2 4acs ( s  b) 2 4abs( s  c)
la2  , lb  , lc 
bc ac ab

y por tanto:
4bcs ( s  a) 4acs ( s  b) 4abs( s  c) 43 a 2b 2c 2 s 3 ( s  a)( s  b)( s  c)
la2lb2lc2     
bc ac ab (b  c)(a  c)(a  b)
43 a 2b 2c 2 s 2 ABC  43 a 2 b 2 c 2 s 2 ( s  r ) 2
2
43 a 2 b 2 c 2 s 4 r 2
  
(b  c)(a  c)(a  b) (b  c)(a  c)(a  b) (b  c)(a  c)(a  b)

Donde hemos aplicado la Fórmula de Heron (GA/10.5.11) y GA/11.4.8. Luego:


8abc
lalblc  s 2r  s 2r
(b  c)(a  c)(a  b)

Donde hemos aplicado la desigualdad del problema 2.3.

4bcs ( s  a)
b) Puesto que 4bc  (b  c)2 (Desigualdad AM-GM) tenemos que la2   s( s  a) , y
bc
de la misma forma
lb2  s(s  b) y lc2  s(s  c) .
y por tanto:
la2  lb2  lc2  s( s  a)  s( s  b)  s( s  c)  s( s  a  s  b  s  c) 
 abc 
 s 3  (a  b  c)   s 2
 2 

c) Basta aplicar el Problema 2.5a, o de una manera directa:


s a s b
la2lb2  s 2 ( s  a)( s  b)  lalb  s ( s  a)( s  b)  s ( s  a)( s  b)  s 
2
2s  a  b c
s s
2 2
En donde hemos aplicado la desiguadad AM-GM, y por tanto:
c a b abc
la lb  lb lc  lc la  s  s  s  s  s2
2 2 2 2

12.18
Puesto que no hay relación entre los puntos M, N y P, vamos a demostrar que
bc ac ab
 2R ,  2R y  2R
AM BN CP
Veamos la primera, las otras dos se demuestran con argumentos similares.

El punto de una recta más próximo a un punto dado se encuentra trazando la perpendicular por
dicho punto, así pues
AM  ha , con ha la altura del triángulo correspondiente al vértice A.
Luego
bc bc
  2R
AM ha

Esta última igualdad se puede demostrar por, por ejemplo:


ABC   a ha  abc  4RABC   abc  4R a ha  bc  2R ha
2 2

Una manera alternativa de resolver este problema podría ser tener en cuenta la observación en
GA/9.2.2
a  AM sin(AMB )
 ABC  
abc bc
  2 R sin(AMB )
4R 2 AM
ca ab
Y de la misma forma  2 R sin(BNA) y  2 R sin( APC )
BN CP
 2Rsin(AMB)  sin(BNA)  sin(PC )  2R  3  6R
bc ca ab
Y por tanto  
AM BN CP

12.19
Primera versión.
Aplicando la desigualdad 4.4
ab  bc  ac  1  1  1   32  1  1  1  9
 ab bc ac  ab bc ac ab  bc  ac
Es suficiente demostrar
9 1
 2  9 R 2  ab  bc  ac
ab  bc  ac R

Pero aplicando la desigualdad del problema 2.5a y la desigualdad de Leibniz (12.4)


ab  bc  ac  a 2  b2  c2  9R2

Tal y como queríamos ver.

Segunda versión.
(a  b  c)r
Aplicamos las identidades conocidas ABC   , ABC   sr 
abc
4R 2
Y la desigualdad de Euler R  2r
1 1 1 c  a  b 2ABC / r 1 1
      2
ab bc ac abc 4 RABC  2rR R

12.20
Sean x  AC , y  BD . Aplicando el Teorema de la bisectriz:
3 y 30
  30  xy  y  (puesto que está claro que x  0 pues es un lado del triángulo)
x 10 x

Aplicando las desigualdades triangulares:


 x  10  3  y  x  13  y  x  13  30 / x
  
3  y  x  10   y  x  7  30 / x  x  7
10  x  3  y 7  x  y 7  x  30 / x
  

 x 2  13x  30  0  x 2  13x  30  x  15x  2  x  0,15


30
x  13 
x
30
 x  7  30  x 2  7 x  30  x 2  7 x  0  0  x 2  7 x  30  ( x  3)( x  10)  x  3
x
30
7 x  7 x  x 2  30  7 x  x 2  30  0  0  x 2  7 x  30
x

Esta última desigualdad se cumple siempre, luego el conjunto solución es el conjunto


intersección de las dos primeras, es decir, 3,15 , y la respuesta es 3  15  18 (C).

12.21
Ver PG/#7.71

13.1
Esta inecuación lleva implícita la condición 1  2x  0  2x  1  x  1/ 2 , y también
x  0 , pues x  0 anularía el denominador de la división de la parte izquierda.

Multiplicamos y dividimos la parte izquierda por el conjugado para eliminar la raíz:


  
1  1  2 x 1  1  2 x  12  (1  2 x)  1  1  2 x  2 x 
1  1  2 x  1 
2

1  2 x   2 x   4 x 2 
2 2

4 x 1  1  2 x     1  
2 2
4x 2 2
4x2 1  1  2x 2
  1  2x
1  1  2 x  1  1  2 x  1  1  2 x 
2 2 2
4x2

Luego la inecuación del enunciado es equivalente a:


1  2
1  2x  2x  9  1  2 1  2x  1  2x  2x  9 
2 1  2x  7  1  2x  7 / 2 
1  2 x  7 / 2  49 / 4  2 x  49 / 4  1  45 / 4  x  45 / 8
2

1 45
Así pues, el resultado es x y x0
2 8

13.2
Esta inecuación lleva implícitas dos condiciones:
3 x  0  3 x 
  1  x  3
x  1  0  x  1
Tomando f ( x)  3  x  x  1 , vemos que esta función es decreciente en el intervalo
 1  x  3 , y que toma valores desde f (1)  2 hasta f (3)  2 . Luego existirá un único
valor a tal que f (a)  1/ 2 , y el intervalo buscado será 1  x  a .

Observamos que f (1)  0 , luego a  1 .

1
El problema se reduce a resolver la ecuación 3  a  a 1 
, que elevando al cuadrado se
2
33 31
convierte en la ecuación de segundo grado a 2  2a   0 , cuyas raíces son a  1  .
64 8
31
Puesto que sabemos que a  1 , la única solución válida es a  1  y la solución del
8
31
problema es  1  x  1  .
8
Fuente de esta solución: International Mathematical Olympiads 1959-1977 Compiled and with solutions by Samuel L.Greitzer, pág. 46.

Nota: Si intentamos resolver este mismo problema por métodos digamos más convencionales,
vemos que aparece en el resultado un intervalo no aceptable:

 
2
1 1 2
 3  x  x 1     3  x  x 1  3  x  x 1 2 3  x x 1 
2 2
 15
 4  2 3  x x  1   2 3  x x  1  2 3  x x  1 
1 15

4 4 4
2 2

3  x x  1  15  3  x x  1   15   3  2 x  x 2   15  
8 8 8
2
 15  33
3  2x  x2       2x  x2  0
8 64
Para resolver esta última inecuación determinamos las raíces de la ecuación cuadrática:
33 31
  2x  x2  0  x  1 
64 8
33 31 31
Luego   2x  x2  0  x  1   x 1
64 8 8
31
Sin embargo, el intervalo x  1  no es solución de la inecuación original.
8

13.3
1
Dominio de definición: La inecuación no está definida en x  b y x  .
b
ax  1 x  a
 
ax  1 x  a
 0
ax  1x  b  bx  1x  a   0 
bx  1 x  b bx  1 x  b bx  1x  b
ax 2  bx 2  a  b
0
 
a  b x 2  1  0  x 2  1  0
bx  1x  b bx  1x  b bx  1x  b
En donde hemos tenido en cuenta que a  b  1 . Dividir entre b no cambia el signo, y por tanto:

x2 1 ( x  1)( x  1)
0 0
bx  1x  b  1
 x   x  b 
 b

Se cumplirá cuando el numerador y el denominador tengan el mismo signo. El numerador es


negativo en  1  x  1 , y el denominador es negativo en  b  x  1 / b .

El conjunto solución es  ,b   1,1 / b  1, .

14.1
De la ecuación x 4  y 4  1 se deduce que x  1 .
En efecto, 1  x  1  x 4  x 4  y 4  1 , absurdo. De la misma forma se demuestra que y  1 .
Está claro, por otro lado, que x  1, y  0 ; x  0, y  1 son soluciones del sistema. Veamos que
no hay ninguna más.

Supongamos que  1  x  0 es solución del sistema. Entonces:


 1  x  0  1  x3  0  y3  1  x3  1  y  1 absurdo.

Supongamos ahora que 0  x  1 es solución del sistema. Entonces:


0  x  1  0  x3  1  0  y 3  1  x3  1  0  y  1
Y por tanto:

0  x  1  x 4  x3 
3
 1  x 4  y 4  x3  y 3  1 absurdo.
0  y 1 y  y 
4

Así pues, las dos soluciones anteriores son las únicas soluciones del sistema.

Fuente de esta solución: Practice problems for the Math Olympiad P. Gracia, D.Klein, L.Luxemburg, L. Qiu, J. Szucs.
Algunos apuntes biográficos.
Jakob Bernoulli (Basilea (Suiza), 27 de diciembre de 1654 16 de agosto de 1705), más
conocido como Jacob Bernoulli por la traducción de su nombre al alemán, fue un
matemático y científico ruso. Por deseo de su padre estudió Filosofía y Teología,
aunque aprovechó su estancia en la Universidad para iniciarse en Matemáticas, que era
su verdadera vocación.

Permitió el avance de la teoría de la probabilidad y las aplicaciones del cálculo. En su


primer artículo sobre series infinitas (1689) presentó la llamada Desigualdad de
Bernoulli. Se interesó en las propiedades de curvas como la catenaria, la tractriz, la
isocrona y la espiral logarítmica. En 1960 usó por primera vez la palabra integral,
aunque Leibniz prefirió el término summatorius. Contribuyó al estudio de la ecuación
que hoy lleva su nombre y describió la llamada lemniscata de Bernoulli. Su obra más
destacable fue Ars Conjectandi, publicada en 1713 con carácter póstumo. En ella se
incluye el concepto de números de Bernoulli y el principio básico de teoría de
probabilidad: el Teorema de Bernoulli, conocido en la actualidad como Ley débil de los
grandes números.

Titu Andreescu (Rumanía, 1956), profesor en la Universidad de Texas, es conocido


por su destacable trabajo y participación en olimpiadas internacionales de matemáticas.
Ha llegado a ser director en las olimpiadas AMC y MOP, entrenador principal de las
IMO estadounidenses y presidente de USAMO. Ha publicado numerosos libros sobre
problemas y ejercicios de olimpiadas matemáticas, entre los que destaca la desigualdad
que trataremos a continuación, conocida como el Lema de Titu.

Otto Ludwing Hölder (Stuttgart, 22 de diciembre de 1859 – Leipzig, 29 de agosto de


1937) fue un matemático alemán. Estudió Ingeniería, durante un año, en el Politécnico
de Stuttgart y desde 1877 estudió en la Universidad de Berlín. En 1882 presentó su tesis
doctoral en la Universidad de Tübingen y trabajó en la Universidad de Leipzig desde
1899 hasta su jubilación.

Es conocido por sus contribuciones tanto en álgebra como en análisis matemático. Se


interesó por el álgebra y, en concreto por la teoría de grupos, influenciado por
Kronecker y Klein. Probó el conocido Teorema de Jordan-Hölder, investigó los grupos
finitos simples y publicó un artículo mostrando que los grupos finitos simples hasta
orden 200 eran conocidos. En 1895 redactó un extenso trabajo sobre extensiones de
grupos e introdujo los conceptos de automorfismo interno y externo.
Fuente de las notas biográficas: Cristina Bermejo Moreno (Desigualdades matemáticas clásicas, 2017)

También podría gustarte